You are on page 1of 74

Total Marks : 200.

00
Prelims Test 22 - GS Test 15
( Insta Prelims Test Series 2024 7.0 ) Mark Scored : 83.54

1. Consider the following statements:


1. Coal deposits in India contain high levels of ash as compared to other coal-mining countries
including China.
2. Burning coal with more ash can lead to erosion and the failure of boiler tubes and its
components.
3. Unlike China, India has not formulated any coking coal strategy to reduce its coal imports.
4. The major overseas supplier countries of coal for India include Indonesia, Australia and Russia.

How many of the statements given above are correct?


A. Only one
B. Only two
C. Only three
D. All four
7
Your Answer : C 277
6 3
Correct Answer : C
560
- 9
Answer Justification : m
il.co
a 2031-2032, India’s national
According to Central Electricity Authority (CEA) projections for FY
m
g
4; energy sources by progressively
grid can absorb 924 TWh of electricity from various renewable
6
adding 47 GW of battery storage capacity and 27 GW#of pumped storage projects by FY 32.
&
a95
t v generally contain high levels of ash (35-50%)
Statement 1 is correct: Coal deposits inaIndia
s
compared to those mined in other major
r i va coal-mining countries, like Australia, China, and the
U.S.
u tis
r
hBurning
s
a - components, affecting the plant’s availability, efficiency, and
Statement 2 is correct: coal with more ash leads to the erosion and eventual failure

ta
of boiler tubes and vother
performance.as
S riv
i
Any
r utmajor increase in battery storage capacity in India will require the import of critical
Shminerals like lithium, cobalt, nickel, and graphite, which are controlled by other countries
(mainly China), posing significant risks to India’s energy security.

Statement 3 is not correct: With a focus on Aatmanirbhar Bharat, the Ministry of Coal has
formulated a coking coal strategy to enhance coking coal availability in the country to reduce
imports.

Statement 4 is correct: The 5 major overseas supplier countries of coal are Indonesia,
Australia, South Africa, USA & Russia.

2. Consider the following statements:


The Central Pollution Control Board-
1. Was constituted under the Water (Prevention and Control of Pollution) Act, 1974.
2. Is entrusted with the powers under the Air (Prevention and Control of Pollution) Act, 1981.
instacourses.insightsonindia.com 1
© Insights Active Learning | All rights reserved - 18107. You may not reproduce, distribute or exploit the contents in any
form without written permission by copyright owner. Copyright infringers may face civil and criminal liability
Total Marks : 200.00
Prelims Test 22 - GS Test 15
( Insta Prelims Test Series 2024 7.0 ) Mark Scored : 83.54

3. Serves as a field formation under the provisions of the Environment (Protection) Act, 1986.

How many of the statements given above are correct?


A. Only one
B. Only two
C. All three
D. None

Your Answer : C
Correct Answer : C

Answer Justification :

Option (c) is correct: The Central Pollution Control Board (CPCB), statutory organisation, was
7
constituted in September, 1974 under the Water (Prevention and Control of Pollution) Act, 1974.
277
3
06 and
Further, CPCB was entrusted with the powers and functions under the Air (Prevention
6
Control of Pollution) Act, 1981. 95 -
It serves as a field formation and also provides technical services to.cthe
m
o Ministry of Environment
and Forests of the provisions of the Environment (Protection) Act, i l
a 1986.
;g m
6 4(Prevention
& #
Principal Functions of the CPCB, as spelt out in the Water and Control of Pollution)
Act, 1974, and the Air (Prevention and Control of5Pollution) Act, 1981,
va9
a
t wells in different areas of the States by prevention,
sand
to promote cleanliness of streams
v a
s
control and abatement of water
i ri pollution, and
to improve the qualityruoftair and to prevent, control or abate air pollution in the country.
h
a -s
stav statements:
3. Consider the following
va
1. The idea rofi a Lokpal which stands for the central anti-corruption ombudsman was first
i S
r ut
mentioned in the Constituent Assembly of India.
S
2. h
Maharashtra was the first state to establish a separate Lokayukta body under the Maharashtra
Lokayukta and Upayukta Act.
3. The Lokpal unlike the Lokayukta deals with complaints against public servants, including the
Lokpal chairperson and members.

How many of the statements given above are correct?


A. Only one
B. Only two
C. All three
D. None

Your Answer : A
Correct Answer : A

Answer Justification :

instacourses.insightsonindia.com 2
© Insights Active Learning | All rights reserved - 18107. You may not reproduce, distribute or exploit the contents in any
form without written permission by copyright owner. Copyright infringers may face civil and criminal liability
Total Marks : 200.00
Prelims Test 22 - GS Test 15
( Insta Prelims Test Series 2024 7.0 ) Mark Scored : 83.54

Statement 1 is not correct: The idea of a Lokpal — the central anti-corruption ombudsman —
first came up in 1963 during a discussion on Budget allocation of the Union Law Ministry.

Bills seeking an ombudsman were introduced in Parliament eight times between 1968 and 2001
but were not passed, noted a 2020 report by anti-corruption organisation Transparency
International India.

Statement 2 is correct: Over the years, different states set up their own Lokayuktas — the
state equivalent of the Lokpal. Maharashtra was first in this respect with its Lokayukta body
established in 1971 under the Maharashtra Lokayukta and Upayukta Act.

The Lokpal and Lokayuktas Act, 2013, subsequently came into force on January 16, 2014. The
Act provides for establishing a Lokpal headed by a Chairperson, who is or has been a Chief
Justice of India, or is or has been a judge of the Supreme Court, or an eminent person who fulfils
eligibility criteria as specified.
7
277
3
Statement 3 is not correct: The Lokpal and Lokayukta both deal with complaints6against
6 0
public servants, including the Lokpal chairperson and members. 5
-9
.c om
i l
aForest
4. Consider the following statements regarding the Kyasanur
;g m Disease (KFD) Virus:
4
1. The KFD Virus can cause epizootics with high fatality in6primates.
2. Hard ticks are the reservoir of KFD Virus. 5 &#
3. Mosquitoes are the most common hosts for vKFD a9 Virus.
a
st of South East Asia.
4. The KFD disease is endemic to the countries
va
i sri
How many of the statementsugivent above are correct?
h r
A. Only one
a -s
tav
B. Only two
s
iva
C. Only three
D. All r
four
i S
r ut
ShYour Answer :
Correct Answer : B

Answer Justification :

Statement 1 is correct: Kyasanur Forest disease (KFD) is caused by Kyasanur Forest disease
virus (KFDV), a member of the virus family Flaviviridae. KFDV can cause epizootics with high
fatality in primates.

KFDV was identified in 1957 when it was isolated from a sick monkey from the Kyasanur Forest
in Karnataka (formerly Mysore) State, India. Since then, between 400-500 humans cases per
year have been reported.

Statement 2 is correct: Hard ticks (Hemaphysalis spinigera) are the reservoir of KFD virus
instacourses.insightsonindia.com 3
© Insights Active Learning | All rights reserved - 18107. You may not reproduce, distribute or exploit the contents in any
form without written permission by copyright owner. Copyright infringers may face civil and criminal liability
Total Marks : 200.00
Prelims Test 22 - GS Test 15
( Insta Prelims Test Series 2024 7.0 ) Mark Scored : 83.54

and once infected, remain so for life.

Statement 3 is not correct: Rodents, shrews, and monkeys are common hosts for KFDV after
being bitten by an infected tick.

Statement 4 is not correct: The disease is endemic to 16 districts in 5 states of Southern


India and is reported in the dry season, most commonly in humans travelling to the forests in
these areas.

5. Consider the following statements regarding the ‘Biosimilars’:


1. They provide an effective treatment against the Crohn’s disease.
2. They increase access to lifesaving medications at potentially lower costs.
3. They are used in India in the form of insulin and monoclonal antibodies.
7
277
How many of the statements given above is/are correct? 6 3
A. Only one 560
B. Only two - 9
m
C. All three
il.co
D. None
m a
4 ;g
6
&#
Your Answer : A
5
a9
Correct Answer : C
ta v
Answer Justification :
i vas
tisr
ru are safe and effective treatment options for many illnesses
Option (c) is correct: Biosimilars
h
such as chronic skin and s
- bowel diseases (like psoriasis, irritable bowel syndrome, Crohn’s
v a
a arthritis, kidney conditions, and cancer.
disease and colitis),
st
iv a
i Sr
Biosimilars increase access to lifesaving medications at potentially lower costs.
u t Indian companies are taking multiple steps to involve them in manufacturing and
r
Sh marketing to tap this huge potential.
Biosimilars approved and used in India mainly consist of the vaccines, monoclonal
antibodies, insulin, and recombinant proteins.
India has achieved the distinction of being the second largest supplier of vaccines in the
world.

6. Consider the following statements:


1. It is mandatory for a political party to get registered with the Election Commission to avail itself
the advantage under the Representation of the People Act, 1951.
2. There is no explicit provision in the Representation of People Act of 1951, that provides for the
de-registration of a political party.

Which of the statements given above is/are correct?


A. 1 only

instacourses.insightsonindia.com 4
© Insights Active Learning | All rights reserved - 18107. You may not reproduce, distribute or exploit the contents in any
form without written permission by copyright owner. Copyright infringers may face civil and criminal liability
Total Marks : 200.00
Prelims Test 22 - GS Test 15
( Insta Prelims Test Series 2024 7.0 ) Mark Scored : 83.54

B. 2 only
C. Both 1 and 2
D. Neither 1 nor 2

Your Answer : A
Correct Answer : B

Answer Justification :

Under the constitution, the Election Commission is required to function independently and
ensure free and fair elections.

An enquiry into non-compliance with the conditions for the grant of registration might involve
the Commission in matters of a political nature and could mean monitoring by the Commission
of the political activities, programmes and ideologies of political parties. 7
277
3
6 06
Statement 1 is not correct: It is not mandatory to register with the Election Commission but
95
registering as a political party with the EC has its advantage in terms of intending to avail itself
of the provisions of the Representation of the People Act, 1951, (relating to- registration of
m
political parties). .co il
m a
Statement 2 is correct: The EC contended that at present
4 ;g it only has the power to register a
6
political party. This is laid down in Section 29A of the#Representation of People Act.
9 5&
a
tavunder Section 29A.
In fact, there is no explicit provision in the Representation of People Act, 1951, for
s
va
deregistration of a political party registered
r i
There are nearly 2,800 registeredu tis unrecognised political parties in India. Besides, eight parties
r
hparties
are recognised as national
- s and over 50 as recognised state parties.
a
stav
r iva
S
7. Consider
u ti the following statements regarding the Electronic or e-cigarettes:
r systems heat a liquid to create aerosols that are inhaled by the user.
1. hThese
S
2. These cigarettes contain liquids that are typically non-toxic for the user’s health.
3. The cigarettes do not expose the non-smokers to nicotine or other chemicals.

How many of the statements given above are correct?


A. Only one
B. Only two
C. All three
D. None

Your Answer : B
Correct Answer : A

Answer Justification :

instacourses.insightsonindia.com 5
© Insights Active Learning | All rights reserved - 18107. You may not reproduce, distribute or exploit the contents in any
form without written permission by copyright owner. Copyright infringers may face civil and criminal liability
Total Marks : 200.00
Prelims Test 22 - GS Test 15
( Insta Prelims Test Series 2024 7.0 ) Mark Scored : 83.54

Electronic cigarettes (or e-cigarettes) are the most common form of electronic nicotine delivery
systems (ENDS) and electronic non-nicotine delivery systems (ENNDS) but there are others,
such as e-cigars and e-pipes. ENDS contain varying amounts of nicotine and harmful emissions.

Statement 1 is correct: These systems heat a liquid to create aerosols that are inhaled by the
user.

Statement 2 is not correct: These so-called e-liquids may or may not contain nicotine (but
not tobacco) but also typically contain additives, flavours and chemicals that can be toxic to
people’s health.

Statement 3 is not correct: ENDS use can also expose non-smokers and bystanders to
nicotine and other harmful chemicals.

Electronic delivery systems have also been linked to a number of physical injuries, including
7
burns from explosions or malfunctions, when the products are not of the expected standard
277 or
are tampered with by users. 63
560
- 9
m
8. With reference to Haemophilia, consider the following statements: il .co
a
1. It is caused by the genetic mutation.
4 ;gm
2. A female who is a carrier does not show its symptoms.6
#
5& joint disease.
3. It can result in bleeding within joints causing chronic
va9
How many of the statements given above
a
st are correct?
va
A. Only one
i sri
t
B. Only two
h ru
C. All three
a -s
tav
D. None
a s
Your Answer
S riv : A
i
ut Answer : B
Correct
r
Sh
Answer Justification :

Statement 1 is correct: Hemophilia is caused by a mutation or change, in one of the genes,


that provides instructions for making the clotting factor proteins needed to form a blood clot.

Statement 2 is not correct: A female with one affected X chromosome is a “carrier” of


hemophilia. Sometimes a female who is a carrier can have symptoms of hemophilia.

Statement 3 is correct: Hemophilia can result in:

Bleeding within joints that can lead to chronic joint disease and pain
Bleeding in the head and sometimes in the brain which can cause long term problems,
such as seizures and paralysis

instacourses.insightsonindia.com 6
© Insights Active Learning | All rights reserved - 18107. You may not reproduce, distribute or exploit the contents in any
form without written permission by copyright owner. Copyright infringers may face civil and criminal liability
Total Marks : 200.00
Prelims Test 22 - GS Test 15
( Insta Prelims Test Series 2024 7.0 ) Mark Scored : 83.54

9. Consider the following statements regarding the Baba Raghav Das:


1. He accompanied Mahatma Gandhi during Dandi March in 1930.
2. He is best remembered in Indian history as Poorvanchal’s Gandhi.
3. He took active part in Vinoba Bhave’s Bhoodan movement.

How many of the statements given above is/are not correct?


A. Only one
B. Only two
C. All three
D. None

Your Answer :
Correct Answer : D
7
Answer Justification : 277
3
6 06
Option (d) is correct: Baba Raghav Das from Pune, was the successor of the 5Gaddi (the
-9
religious seat) of Anant Mahaprabhu at Deoria district of erstwhile United Provinces.
.c om
It was Gandhiji who during his visit to Gorakhpur in 1921 a
l
i Raghav Das and called him
met
Baba. ;g m
4
He accompanied Gandhiji during the Dandi March
& #6 (Salt March, 1930). Das was a well-
wisher of the famous revolutionary Ram 9 5 Bismil; a sentiment reflected in a
Prasad
memorial erected by him for Bismil, a v a
t in his ashram.
sprogrammes
Closely following the constructive
i va of Gandhi he used to organize public
r
tis of freedom, social reforms, cleanliness and upliftment of
meetings propagating values
u
dalits. r
sh governance too as a Member of Legislative Assembly in 1948.
He had a stint in-public
t va
Post India’saindependence, he continued his social service and took part in Vinoba
s
a Bhoodan movement for the upliftment of the downtrodden leaving a legacy
rivremembered him as Poorvanchal’s Gandhi.
Bhave’s
i S
which
r ut
Sh
10. Consider the following statements:
1. In a unitary form of Government, the Centre has plenary powers of administration with its
constituent units having little autonomy.
2. In India, the residuary powers of legislation that is the power to make law in a field not specified
in the Constitution is vested in the Parliament.
3. The Sixth Schedule of the Indian Constitution contains provisions about the administration of
tribal areas of Meghalaya, Manipur and Mizoram.

How many of the statements given above are correct?


A. Only one
B. Only two
C. All three
D. None

instacourses.insightsonindia.com 7
© Insights Active Learning | All rights reserved - 18107. You may not reproduce, distribute or exploit the contents in any
form without written permission by copyright owner. Copyright infringers may face civil and criminal liability
Total Marks : 200.00
Prelims Test 22 - GS Test 15
( Insta Prelims Test Series 2024 7.0 ) Mark Scored : 83.54

Your Answer : B
Correct Answer : B

Answer Justification :

Statement 1 is correct: Nations are described as ‘federal’ or ‘unitary’, depending on the way
in which governance is organised. In a unitary set-up, the Centre has plenary powers of
administration and legislation, with its constituent units having little autonomy.

In a federal arrangement, the constituent units are identified on the basis of region or ethnicity,
and conferred varying forms of autonomy or some level of administrative and legislative
powers.

Statement 2 is correct: In India, the residuary powers of legislation, that is the power to make
law in a field not specified in the Constitution, is vested in Parliament, whereas in the U.S.,
7
residuary powers are with the States. 77 2
3
is6limited, and
Further, in fiscal matters, the power of the States to raise their own resources 5
06
there is a good deal of dependency on the Centre for financial assistance. - 9
.c om
Statement 3 is not correct: The Sixth Schedule to the Constitution l
ai contains provisions for the
m
administration of tribal areas in Assam, Meghalaya, Tripuragand Mizoram (not Manipur).
6 4;
These create autonomous districts and autonomous
5 &#regions. Any autonomous district with
9
different Scheduled Tribes will be divided into
ta vaautonomous regions.
i vas
tisr
u
shr
11. Consider the following statements regarding Anarcho-capitalism:
a - of the state and the provision of law and order to be controlled by
1. It calls for the abolition
v
private companies
a sta in a free market.
r v private companies competing in a free market can provide quality legal services
2. It believesithat
S
i
that tare far better than services provided by the state.
r u
ShWhich of the statements given above is/are correct?
A. 1 only
B. 2 only
C. Both 1 and 2
D. Neither 1 nor 2

Your Answer : C
Correct Answer : C

Answer Justification :

Statement 1 is correct: Anarcho-capitalism refers to a political philosophy which calls for the
abolition of the state and for the provision of law and order to be controlled by private
companies in a free market.

instacourses.insightsonindia.com 8
© Insights Active Learning | All rights reserved - 18107. You may not reproduce, distribute or exploit the contents in any
form without written permission by copyright owner. Copyright infringers may face civil and criminal liability
Total Marks : 200.00
Prelims Test 22 - GS Test 15
( Insta Prelims Test Series 2024 7.0 ) Mark Scored : 83.54

The term was coined by American libertarian economist Murray Rothbard, while Belgian
political economist Gustave de Molinari is considered to be the first anarcho-capitalist.
Traditionally, free market advocates have supported the private provision of most goods
and services, except police and courts which, it was believed, could only be provided by
the state.
Statement 2 is correct: But anarcho-capitalists believe that private companies
competing in a free market can provide policing and legal services as well, often far
better than the state when it comes to quality and price.
Just the way private companies today can offer better cars, telephone services, etc. at
cheaper prices than the government, anarcho-capitalists argue, private companies can
also provide a far better police and legal system.

12. With reference to Poland, consider the following statements:


1. It is a Central European country. 7
2. It is bordered in the north by the Aegean Sea. 277
6 3
3. It shares its maritime borders with Denmark and Sweden.
560
- 9
How many of the statements given above are not correct?
.c om
A. Only one
ai l
B. Only two
;g m
C. All three 6 4
D. None &# 5
va9
ta
as
Your Answer : A
i v
sr
Correct Answer : A
i
r ut
Answer Justification : sh
v a-
Statement 1 sista correct: Poland is a Central European country. It is positioned both in the
a
riv Eastern hemispheres of the Earth.
Northern and
S
i
r ut is bordered by 7 nations: by Germany in the west; the Czech Republic in the southwest;
Poland
ShSlovakia in the south; Ukraine in the southeast; Belarus in the east; Lithuania and Russia in the
northeast.

Statement 2 is not correct: Poland is bordered in the north by the Baltic Sea.

Statement 3 is correct: The country shares its maritime borders with Denmark and Sweden.

13. Consider the following statements:


The Aerosols-
1. Include tiny particles like smoke, dust, and pollutants.
2. Can absorb and scatter sunlight.
3. Are being analysed under the PACE mission of NASA.

instacourses.insightsonindia.com 9
© Insights Active Learning | All rights reserved - 18107. You may not reproduce, distribute or exploit the contents in any
form without written permission by copyright owner. Copyright infringers may face civil and criminal liability
Total Marks : 200.00
Prelims Test 22 - GS Test 15
( Insta Prelims Test Series 2024 7.0 ) Mark Scored : 83.54

How many of the statements given above is/are correct?


A. Only one
B. Only two
C. All three
D. None

Your Answer : C
Correct Answer : C

Answer Justification :

Option (c) is correct: Aerosols, which include tiny particles like smoke, dust, and pollutants,
may seem inconspicuous, but they play a crucial role in influencing our climate.

Aerosols absorb and scatter sunlight, determining the amount of solar energy reaching7
Earth’s surface. 2 77
63 the
The PACE mission of NASA will not only analyze aerosols but also delve into0studying
6
color of the ocean.
- 95
The primary science instrument for PACE is the Ocean Color Instrument
m (OCI), designed to
c o
l. shortwave infrared.
measure the ocean’s color across a spectrum from ultraviolet to
ai
;g m
6 Range separates- 4
14. The Pangolakha Range which extends below the#Chola
5 &
va9
A. West Bengal and Bangladesh ta
B. Manipur and Myanmar i vas
C. Uttarakhand and Nepal tisr
D. Sikkim and Bhutan hr
u
- s
Your Answer : ta va
s
iva : D
Correct Answer
i Sr
t
ru
Answer Justification :
Sh
Option (d) is correct: The Pangolakha Range, extending below the Chola Range, separates
Sikkim from Bhutan.

Hathichirey forms the tri-junction between Bhutan, Sikkim and West Bengal where further
down the forest continues as the Neora Valley National Park.
The Sanctuary has typical alpine-temperate-subtropical vegetation with high-altitude
lakes around Jelep La.
Rhododendron, Silver Fir, Juniper forest and associated ground flora, moss-filled oak
forests with dense bamboo thickets form ideal habitat for the Red Panda Ailurus fulgens,
the State Animal.

15. “The Global status report on road safety 2023” has been recently released by the-

instacourses.insightsonindia.com 10
© Insights Active Learning | All rights reserved - 18107. You may not reproduce, distribute or exploit the contents in any
form without written permission by copyright owner. Copyright infringers may face civil and criminal liability
Total Marks : 200.00
Prelims Test 22 - GS Test 15
( Insta Prelims Test Series 2024 7.0 ) Mark Scored : 83.54

A. World Health Organisation


B. United Nations Development Programme
C. Indian Federation of Road Safety
D. United Nations Industrial Development Organization

Your Answer : B
Correct Answer : A

Answer Justification :

Option (a) is correct: The Global Status Report on road safety 2023 is the 5th report led by
the World Health Organisation.

The Global status report on road safety 2023 details the scale of global road traffic
deaths, and progress in advancing laws, strategies and actions to reduce them around7the
world. 277
3
The fifth report in a series, it provides an overview of progress between 2010
6 06and 2021,
95 2021-2030
and sets a baseline for the United Nations Decade of Action for Road Safety
-
omslightly to 1.19 million
target of halving road traffic deaths and injuries by 2030.
.
The report shows that the number of road traffic deaths haslfallen
i c
ma an impact.
per year, and that efforts to improve road safety are having
;g
6 4
&#
95about in the news in the context of?
16. The term ‘Aktocyte' has been recently talked
ta va
i v
A. The cells located within the centralas nervous system that have physiological properties
related to homeostasis. tis
r
u
B. The tablets designed
shras an adjuvant to cancer radiotherapy marking a significant
advancement a in -cancer care.
v
C. The machine
a sta learning that emulates human brain and solves common problems in
iv Intelligence.
Artificial
Srmetal oxide perovskites that have found applications in catalysis and energy
D. iThe
t
ru conversion.
Sh
Your Answer : B
Correct Answer : B

Answer Justification :

Option (b) is correct: The AKTOCYTE tablets have recently shown remarkable results,
particularly in pelvic cancer patients suffering from radiotherapy-induced Cystitis (Blood in
urine).

Patients treated with AKTOCYTE tablets demonstrated an extraordinary recovery,


eliminating the need for surgical removal of the urinary bladder.
The tablets, designed as an adjuvant to cancer radiotherapy, regenerative nutraceutical,
immunomodulator, and antioxidant, mark a significant advancement in cancer care.
AKTOCYTE has received approval from the Food Safety and Standards Authority of India

instacourses.insightsonindia.com 11
© Insights Active Learning | All rights reserved - 18107. You may not reproduce, distribute or exploit the contents in any
form without written permission by copyright owner. Copyright infringers may face civil and criminal liability
Total Marks : 200.00
Prelims Test 22 - GS Test 15
( Insta Prelims Test Series 2024 7.0 ) Mark Scored : 83.54

(FSSAI), operating under the Ministry of Health & Family Welfare, Government of India.
The tablets serve as more than just a supplement. AKTOCYTE is positioned as an adjuvant
to cancer radiotherapy, a regenerative nutraceutical, an immunomodulator, and an
antioxidant, showcasing its versatility in cancer care.

17. Which one of the following Organisations launched the pioneering initiative called
“Self Help Group Bank Linkage Programme (SHG BLP)”?

A. The Reserve Bank of India


B. The State Financial Corporations
C. The Industrial Development Bank of India
D. The National Bank for Agriculture and Rural Development

Your Answer : D 7
Correct Answer : D 277
6 3
560
Answer Justification :
- 9
m
Option (d) is correct: Self Help Group Bank Linkage Programmel.(SHG
i co BLP) is the pioneering
initiative of Financial Inclusion by NABARD and is well poisedm toadeliver the savings and credit
services at the doorstep of rural poor especially women 4 ;g
through their self-managed institutions
6
called SHGs.
5 &#
Based on the observations of variousav
a9
research studies and an action research project
astof ‘SHG-BLP’ has evolved as a cost-effective
carried out by NABARD, the modelv
ri services to the unreached and underserved poor
i s
mechanism for providing financial
t
households.
h ru
s ‘Panchsutras’ viz. conduct of regular group meetings, regular
The SHGs which -follow
a
tav the group, internal lending based on the demand of members, timely
savings within
s
a of loan and maintenance of proper books of accounts are considered to be of
rivquality and over years have proved themselves to be good customers of Banks.
repayment
i S
good
r ut
Sh
18. Consider the following statements regarding the United Nations Convention against
Corruption:
1. It is the only legally binding universal anti-corruption instrument that includes trading in
influence as a form of corruption.
2. India is not a signatory member of the Convention as it has its own asset recovery and anti-
bribery mechanism.

Which of the statements given above is/are correct?


A. 1 only
B. 2 only
C. Both 1 and 2
D. Neither 1 nor 2

Your Answer : A

instacourses.insightsonindia.com 12
© Insights Active Learning | All rights reserved - 18107. You may not reproduce, distribute or exploit the contents in any
form without written permission by copyright owner. Copyright infringers may face civil and criminal liability
Total Marks : 200.00
Prelims Test 22 - GS Test 15
( Insta Prelims Test Series 2024 7.0 ) Mark Scored : 83.54

Correct Answer : A

Answer Justification :

Statement 1 is correct: The United Nations Convention against Corruption is the only legally
binding universal anti-corruption instrument.

The Convention's far-reaching approach and the mandatory character of many of its
provisions make it a unique tool for developing a comprehensive response to a global
problem.
The Convention covers five main areas: preventive measures, criminalization and law
enforcement, international cooperation, asset recovery, and technical assistance and
information exchange.
The Convention covers many different forms of corruption, such as bribery, trading in
influence, abuse of functions, and various acts of corruption in the private sector. 7
Statement 2 is not correct: In May 2011, the Indian Government ratified two UN 2 77
3
Conventions anf became the member of- the United Nations Convention against
6 06
5 Organised
Corruption (UNCAC) and the United Nations Convention against Transnational
Crime (UNTOC) and its three protocols. -9
.c om
m ail
19. Consider the following statements:
4 ;g
1. The Gandak River is a trans-boundary river between # 6 and India.
Nepal
&
95 in the River Gandak.
2. The largest population of Indian Gharials is present
a
a v
3. The river bed cultivation has revived the tgharial population in India.
i vas
sr
ti above are not correct?
How many of the statements given
u
A. Only one
shr
B. Only two
a-
C. All three tav
D. Noneiva
s
i Sr
t
ru Answer : B
Your
ShCorrect Answer : B
Answer Justification :

Statement 1 is correct and statement 2 is not correct: Two recent surveys indicate that
the Gandak, a trans-boundary river between Nepal and India, houses 210 gharials in the Indian
section, the second-largest such population in the country after the Chambal.

The gharial is “Critically Endangered” as per the IUCN Species Survival Commission assessment
in 2007. Its distribution range has shrunk and currently, it occupies only about 2 per cent of its
former range.

Statement 3 is not correct: River bed cultivation threatens gharial survival by alienating
them from the terrestrial component of its habitat leading to desertion and migration. Removal
of sand from riverbanks disrupts gharial behaviour and may even force local populations to

instacourses.insightsonindia.com 13
© Insights Active Learning | All rights reserved - 18107. You may not reproduce, distribute or exploit the contents in any
form without written permission by copyright owner. Copyright infringers may face civil and criminal liability
Total Marks : 200.00
Prelims Test 22 - GS Test 15
( Insta Prelims Test Series 2024 7.0 ) Mark Scored : 83.54

desert the area.

20. Which one of the following countries is not a part of Five Eyes Alliance?

A. Canada
B. Australia
C. New Zealand
D. India

Your Answer : D
Correct Answer : D

Answer Justification : 7
277
3
06with
Option (d) is correct: The United States of America and the United Kingdom along
6
Canada, Australia and New Zealand, form the Five Eyes Alliance. 95 -
m
These partner countries share a broad range of intelligence with
i l .co one another in one of the
world’s most unified multilateral arrangements.
m a
;g
The Five Eyes agreement stands out from other arrangements because the parties are
diverse societies, governed by rule of law and # 64 human rights and are bonded by a
robust
&
common language.
a 95
These characteristics aid the partners
s tavin sharing information with one another to protect
their shared national interests.”
r i va
u tis
shr
a - Alliance (GRCA) has been-
21. Consider the following statements:
The Global River Cities
a v
1. Initiated by the a stGanga-Mekong Co-operation.
r iatv the United Nations Climate Change Conference.
i Sby the International Monetary Fund.
2. Launched
t
ru
3. Funded
Sh
How many of the statements given above are correct?
A. Only one
B. Only two
C. All three
D. None

Your Answer :
Correct Answer : A

Answer Justification :

Option (a) is correct: The Global River Cities Alliance (GRCA) was led by the National
Mission for Clean Ganga (NMCG) under the Ministry of Jal Shakti, Government of India,
was launched at COP28, the United Nations Climate Change Conference in Dubai,

instacourses.insightsonindia.com 14
© Insights Active Learning | All rights reserved - 18107. You may not reproduce, distribute or exploit the contents in any
form without written permission by copyright owner. Copyright infringers may face civil and criminal liability
Total Marks : 200.00
Prelims Test 22 - GS Test 15
( Insta Prelims Test Series 2024 7.0 ) Mark Scored : 83.54

United Arab Emirates.

It included countries namely India, Egypt, Netherlands, Denmark, Ghana, Australia,


Bhutan, Cambodia, Japan and river-cities of The Hague (Den Haag) from the Netherlands,
Adelaide from Australia, and Szolnok of Hungary.
It included International funding agencies the World Bank, Asian Development
Bank (ADB), Asian Infrastructure Investment Bank (AIIB) and knowledge
management institution like KPMG entering into a partnership, widely expanding the
reach of the existing River Cities Alliance (RCA), formed by NMCG in association with
National Institute of Urban Affairs (NIUA) in 2021.
GRCA is a unique alliance covering 275+ global river-cities in 11 countries, international
funding agencies and knowledge management partners and is first of its kind in the world.

22. Consider the following statements regarding the Leishmaniasis Disease: 7


2
1. It is caused by the protozoan parasites which are transmitted by the bite of infected female77
3
phlebotomine sandflies.
6 06
2. It has adversely affected the poorest nations associated with the malnutrition
- 95and population
displacement. m
oco-infection
3. Antiretroviral treatment reduces the development of Leishmania-HIV
i l .c and increases
the rate of survival. m a
4 ;g
Which of the statements given above is/are correct? #
6
5 &
A. 1 only
va9
B. 1 and 2 only
s ta
C. 2 and 3 only iva
i sr
D. 1, 2 and 3
r ut
- sh
va
Your Answer : D
Correct Answerst: a
D
a
S riv
ti Justification :
Answer
u
r
ShOption (d) is correct: There are 3 main forms of leishmaniases: visceral (the most serious
form because it is almost always fatal without treatment), cutaneous (the most common,
usually causing skin ulcers), and mucocutaneous (affecting mouth, nose and throat).

Leishmaniasis is caused by protozoan parasites which are transmitted by the bite of


infected female phlebotomine sandflies.
The disease affects some of the world’s poorest people and is associated with
malnutrition, population displacement, poor housing, a weak immune system and lack of
financial resources.
People living with HIV and who are infected with leishmaniasis have high chances of
developing the full-blown disease, high relapse and mortality rates.
Antiretroviral treatment reduces the development of the disease, delays relapses and
increases the survival.

instacourses.insightsonindia.com 15
© Insights Active Learning | All rights reserved - 18107. You may not reproduce, distribute or exploit the contents in any
form without written permission by copyright owner. Copyright infringers may face civil and criminal liability
Total Marks : 200.00
Prelims Test 22 - GS Test 15
( Insta Prelims Test Series 2024 7.0 ) Mark Scored : 83.54

23. Consider the following statements regarding the Katabatic winds:


1. They carry high-density air from higher elevations down the slope.
2. Examples of katabatic winds include Oroshi winds of Japan.
3. The descending katabatic winds have low relative humidity.

How many of the statements given above are correct?


A. Only one
B. Only two
C. All three
D. None

Your Answer : B
Correct Answer : C
7
Answer Justification : 277
3
6 06
Statement 1 is correct: Unlike anabatic wind which is an upslope wind, katabatic
- 95 winds are
downslope winds. In a simple definition, these are winds that carry high-density
m air from higher
c o
elevations down the slope. The name “katabatic” is derived from the
a i l. Greek word “katabasis”
which means descending. Such downslope winds are sometimes m referred to as fall winds.
4 ;g
Statement 2 is correct: Other examples of katabatic # 6winds include bora winds of the Adriatic,
5 &
oroshi winds of Japan, and piteraq in Greenland.
a9 It is important to note that not all downslope
v
sta
winds are katabatic winds.
va
Statement 3 is correct: They iare s rimost intense as the low-pressure area approaches the
u t in Antarctica, the snow may be scoured away by the force of
hr
coast. In some areas, especially
s
a -
the winds, leading to the formation of Antarctica oases. The descending katabatic winds have
v
sta
low relative humidity.
a
S riv
u ti
r
Sh
24. Consider the following statements regarding the Dnieper River:
1. It originates in the Valdai Hills of Russia.
2. It flows entirely in the Russian Far East Region.
3. It drains its water into the Black Sea.

How many of the statements given above is/are correct?


A. Only one
B. Only two
C. All three
D. None

Your Answer :
Correct Answer : B

Answer Justification :

instacourses.insightsonindia.com 16
© Insights Active Learning | All rights reserved - 18107. You may not reproduce, distribute or exploit the contents in any
form without written permission by copyright owner. Copyright infringers may face civil and criminal liability
Total Marks : 200.00
Prelims Test 22 - GS Test 15
( Insta Prelims Test Series 2024 7.0 ) Mark Scored : 83.54

Option (b) is correct: The Dnieper River is the fourth longest river in Europe. It runs a total
length of 1,368 miles extending from the uplands of Russia’s Valdai Hills where it flows in a
southerly direction through western Russia, Belarus, and Ukraine before emptying into the Black
Sea.

The source of the Dnieper River can be traced back to Russia’s Valdai Hills which rise to an
elevation of 720 feet. The river originates from a diminutive peat bog located on the hill’s
southern slope.

25. The Galathea Bay recently seen in the news is located in-

A. The Great Socotra Island


B. The Chagos Archipelago 7
C. The Indonesian island of Sumatra 277
D. The Great Nicobar Island 6 3
560
- 9
Your Answer :
m
Correct Answer : D
il.co
m a
Answer Justification : 4 ;g
6
Option (d) is correct: The Union Minister for 9 Ports, &#
5 Shipping and Waterways (MoPSW) and
va
sta
Ayush recently visited the site of the proposed International Container Transhipment Port
(ICTP), at Galathea Bay, Great Nicobarva Island.
i sri
t
ru Container Transhipment Port (ICTP) project has reached
The proposed International
h
- s solidifying its position as a transformative initiative with a total
significant milestones,
a
st av of about ₹44,000 Crores.
estimated cost

r i va is strategically important for the country and crucial for the economic and
The project

iS
infrastructural development of the entire region, has garnered key approvals and support
r ut from government bodies.
Sh
26. Consider the following statements:
1. Biochemical oxygen demand represents the amount of oxygen consumed by microorganisms
while they decompose organic matter under aerobic conditions at a specified temperature.
2. Hot summer temperatures and introduction of excess fertilizers to a water body can lessen the
amount of dissolved oxygen in a water body.

Which of the statements given above is/are correct?


A. 1 only
B. 2 only
C. Both 1 and 2
D. Neither 1 nor 2

Your Answer : C

instacourses.insightsonindia.com 17
© Insights Active Learning | All rights reserved - 18107. You may not reproduce, distribute or exploit the contents in any
form without written permission by copyright owner. Copyright infringers may face civil and criminal liability
Total Marks : 200.00
Prelims Test 22 - GS Test 15
( Insta Prelims Test Series 2024 7.0 ) Mark Scored : 83.54

Correct Answer : C

Answer Justification :

Statement 1 is correct: Biochemical oxygen demand (BOD) represents the amount of oxygen
consumed by bacteria and other microorganisms while they decompose organic matter under
aerobic (oxygen is present) conditions at a specified temperature.

Statement 2 is correct: Certain environmental stresses (hot summer temperatures) and other
human-induced factors (introduction of excess fertilizers to a water body) can lessen the
amount of dissolved oxygen in a water body, resulting in stresses on the local aquatic life.

One water analysis that is utilized in order to better understand the effect of bacteria and other
microorganisms on the amount of oxygen they consume as they decompose organic matter
under aerobic (oxygen is present) is the measure of biochemical oxygen demand (BOD).
7
2 77
Determining how organic matter affects the concentration of dissolved oxygen in a 6 3 or
stream
6 0
lake is integral to water-quality management 5
-9
.c om
ai l
27. Consider the following statements: m
1. The Gulf of Aden is an extension of the Mediterranean 6 4;glocated between the Arabian
Sea
Peninsula and the African continent. 5 &#
2. The Gulf of Aden is bounded by the Somali and va9the Socotra Islands in the north and the Arabian
Sea in the east. sta
va
i sri
Which of the statements given
r utabove is/are correct?
h
A. 1 only
a -s
B. 2 only v
sta2
C. Both 1 aand
riv 1 nor 2
D. Neither
S
i
r ut
ShYour Answer : C
Correct Answer : D

Answer Justification :

Statement 1 is not correct: The Gulf of Aden is an extension of the Indian Ocean, tucked
between the Arabian Peninsula and the African continent.

The gulf connects the Red Sea to the Arabian Sea via the Strait of Bab el Mandeb.

The Gulf of Aden is also a critical part of the Suez Canal shipping route, which connects the Red
Sea and the Mediterranean Sea.

Statement 2 is not correct: The Gulf of Aden is located between the Arabian Peninsula
(north) and the Horn of Africa (south). It is bounded to the south by Somali and the Socotra

instacourses.insightsonindia.com 18
© Insights Active Learning | All rights reserved - 18107. You may not reproduce, distribute or exploit the contents in any
form without written permission by copyright owner. Copyright infringers may face civil and criminal liability
Total Marks : 200.00
Prelims Test 22 - GS Test 15
( Insta Prelims Test Series 2024 7.0 ) Mark Scored : 83.54

Islands, north by Yemen, east by the Arabian Sea, and west by Djibouti.

The gulf is connected to the Somali Sea to the south by the Guardafui Channel, and to the Red
Sea on the west by the Strait of Bab el Mandeb.

28. Consider the following statements:


1. Aspartame is an artificial sweetener used in beverage and dairy products including chewable
vitamins.
2. The World Health Organization (WHO) has recently declared aspartame as a possible
carcinogen.

Which of the statements given above is/are correct?


A. 1 only 7
B. 2 only 277
C. Both 1 and 2 6 3
D. Neither 1 nor 2 560
- 9
m
Your Answer : C
il.co
Correct Answer : C
m a
4 ;g
6
Answer Justification :
&#
a 95
Option (c) is correct: Aspartame is present v
a in a wide range of food products – sugar-free diet
soda, ice tea, ice cream, low fat yoghurt, astcereals, and medicines such as chewable vitamins.
r i v
is
ut consuming at least some amount of aspartame in our diet.
In fact, it is quite difficult toravoid
- sh
t a va
The World Health Organization (WHO) recently declared aspartame, an artificial sweetener used
a s
in the food industry, as a possible carcinogen. Substances that potentially cause cancer are
i v
Sr
called carcinogens.
i
t
h ru
S
29. With reference to the Thira Dance, consider the following statements:
1. It is a traditional ritualistic art form performed in Kerala.
2. Its origin is associated with the martial art form, Kalaripayattu.
3. It has a special feature that it is not based on the mythological stories.

How many of the statements given above is/are correct?


A. Only one
B. Only two
C. All three
D. None

Your Answer : B
Correct Answer : B

instacourses.insightsonindia.com 19
© Insights Active Learning | All rights reserved - 18107. You may not reproduce, distribute or exploit the contents in any
form without written permission by copyright owner. Copyright infringers may face civil and criminal liability
Total Marks : 200.00
Prelims Test 22 - GS Test 15
( Insta Prelims Test Series 2024 7.0 ) Mark Scored : 83.54

Answer Justification :

Statement 1 is correct: Thira is a traditional ritualistic art form performed in Kerala, India. It
is primarily performed during the annual festival of Bhagavathy temples, especially in the
Malabar region.

Thira is a significant cultural and religious event that combines elements of dance, drama, and
ritualistic practices. The word "Thira" translates to "dance" in the Malayalam language.

Statement 2 is correct: It is believed to have originated from the traditional martial art form
called Kalaripayattu, which has influenced the movements and choreography of Thira.

The performances are usually conducted by a group of men known as Thira performers or
Thirakkars, who undergo rigorous training in the art form.

Thira performances revolve around mythological and historical stories, often related to the77
7
triumph of good over evil. 32
6 06
5
Statement 3 is not correct: The main highlight of Thira is the portrayal -of9mythological
characters, particularly the goddess Bhadrakali and her battle againstothe m demon Darika.
l .c
m ai
4 ;g
30. Consider the following statements:
& #6
1. A unicorn company is a privately held startupawith 95 a value of over $1 billion.
v
a sta
2. India has the highest number of unicorn companies worldwide.
3. Founders prefer to keep the unicorns
r i vprivate to retain their control over them.
ti s
r u
How many of the statements
- sh given above are not correct?
A. Only one
va
B. Only twosta
C. All r iva
three
S
D. iNone
r ut
Sh Your Answer : B
Correct Answer : A

Answer Justification :

Option (a) is correct: India ranks third with 72 unicorn companies, with a total valuation of
$195.75 billion.

The country’s unicorns make a substantial 5 per cent contribution to the total global unicorn
valuation

Founders who want to retain control tend to keep their unicorns private. But this limits
the potential for growth. And they often must find ways to provide funders with a return on their
investments.

instacourses.insightsonindia.com 20
© Insights Active Learning | All rights reserved - 18107. You may not reproduce, distribute or exploit the contents in any
form without written permission by copyright owner. Copyright infringers may face civil and criminal liability
Total Marks : 200.00
Prelims Test 22 - GS Test 15
( Insta Prelims Test Series 2024 7.0 ) Mark Scored : 83.54

The US has the highest number of unicorn companies worldwide with an impressive 668 - over
three times more than China in second place (172).

31. Consider the following statements:


1. The Indian Constitution provides for a parliamentary form of government both at the Centre and
in the states.
2. The presidential government has a fixed executive system of government and is prevalent in
Russia and Sri Lanka.
3. The parliamentary government is a non-responsible government because it is continuously
subjected to dissolution.

How many of the statements given above are correct?


A. Only one 7
B. Only two 277
6 3
C. All three
560
D. None
- 9
m
Your Answer : B
il.co
Correct Answer : B m a
4 ;g
6
Answer Justification : &#
a 95
Statement 1 is correct: The Constitution v
taof India provides for a parliamentary form of
a s
government, both at the Centre andivin the states. Articles 74 and 75 deal with the
tisr and Articles 163 and 164 in the states.
parliamentary system at the Centre
u
s hr
Statement 2 is correct:
a - The presidential government is also known as non-responsible or
v
ta or fixed executive system of government and is prevalent in USA, Brazil,
non-parliamentary
s
a among others.
riv
Russia, Sri Lanka
i S
r ut
Statement 3 is not correct: The parliamentary government is also known as ‘responsible
h
S government’ as the cabinet (the real executive) is accountable to the Parliament.

32. Which of the following is/are the features of the Parliamentary Government in India?
1. President is both the head of the State and the head of the government.
2. The doctrine of separation of powers is the basis of parliamentary government.
3. The prime minister can advise the President to dissolve the Parliament.

Select the correct answer using the code given below:


A. 1 only
B. 3 only
C. 2 and 3 only
D. 1, 2 and 3

instacourses.insightsonindia.com 21
© Insights Active Learning | All rights reserved - 18107. You may not reproduce, distribute or exploit the contents in any
form without written permission by copyright owner. Copyright infringers may face civil and criminal liability
Total Marks : 200.00
Prelims Test 22 - GS Test 15
( Insta Prelims Test Series 2024 7.0 ) Mark Scored : 83.54

Your Answer : B
Correct Answer : B

Answer Justification :

Statement 1 is not correct: The American (presidential form of government) President is


both the head of the State and the head of government. As the head of State, he occupies a
ceremonial position. As the head of government, he leads the executive organ of government.

Statement 2 is not correct: The doctrine of separation of powers is the basis of the American
presidential system. The legislative, executive and judicial powers of the government are
separated and vested in the three independent organs of the government.

Statement 3 is correct: The lower house of the Parliament (Lok Sabha) can be dissolved by
the President on recommendation of the Prime Minister. In other words, the prime minister can
7
77
advise the President to dissolve the Lok Sabha before the expiry of its term and hold fresh
2
elections. 63
560
- 9
m
33. Consider the following statements: il .co
a
;gm system formed by the
1. The Shadow Cabinet is a unique institution of the Indian cabinet
4
opposition party to balance the ruling cabinet.
& #6
9 5
2. The ministers in India, unlike any other country are required to countersign the official acts of
the Head of the State. v a
a sta
s
Which of the statements given abover iv is/are correct?
i
A. 1 only r ut
B. 2 only - sh
C. Both 1 and 2va
ta
D. Neithera1s nor 2
S riv
i
utAnswer : D
Your
r
ShCorrect Answer : D
Answer Justification :

Option (d) is correct: ‘Shadow cabinet’ is an unique institution of the British cabinet system.
It is formed by the opposition party to balance the ruling cabinet and to prepare its members for
future ministerial office. There is no such institution in India.

Britain has the system of legal responsibility of the minister while India has no such system.
Unlike in Britain, the ministers in India are not required to countersign the official acts of the
Head of the State.

34. Consider the following statements:

instacourses.insightsonindia.com 22
© Insights Active Learning | All rights reserved - 18107. You may not reproduce, distribute or exploit the contents in any
form without written permission by copyright owner. Copyright infringers may face civil and criminal liability
Total Marks : 200.00
Prelims Test 22 - GS Test 15
( Insta Prelims Test Series 2024 7.0 ) Mark Scored : 83.54

1. Unlike the Indian Parliament, the British system is based on the doctrine of the sovereignty of
Parliament.
2. Unlike the Indian Parliament, the British Prime Minister can be a member of any of the two
Houses of Parliament

Which of the statements given above is/are not correct?


A. 1 only
B. 2 only
C. Both 1 and 2
D. Neither 1 nor 2

Your Answer : B
Correct Answer : B

7
77
Answer Justification :
3 2
06 of
Option (b) is correct: The British system is based on the doctrine of the sovereignty
6
95 restricted
Parliament, while the Parliament is not supreme in India and enjoys limited and
-
mfundamental rights.
powers due to a written Constitution, federal system, judicial review and
i l .co
In Britain, the prime minister should be a member of the Lower m aHouse (House of Commons) of
g
4; of any of the two Houses of
the Parliament. In India, the prime minister may be a member
6
Parliament.
5 &#
va9
sta
va
35. Consider the following statements:i sri
t
ruthrough an agreement between the various units.
1. A federation is a state formed
h
- sprovides for a federal system of government.
2. The Constitution of India
a
tavsystem is based on the ‘Canadian model.
3. The Indian federal
s
a
riv
4. The term ‘federation’ has nowhere been used in the Indian Constitution.
i S
r
Howutmany of the statements given above are correct?
Sh A. Only one
B. Only two
C. Only three
D. All four

Your Answer : C
Correct Answer : D

Answer Justification :

Option (d) is correct: A federation is a new state (political system) which is formed through a
treaty or an agreement between the various units.

The Constitution of India provides for a federal system of government in the country. The
framers adopted the federal system due to two main reasons —the large size of the

instacourses.insightsonindia.com 23
© Insights Active Learning | All rights reserved - 18107. You may not reproduce, distribute or exploit the contents in any
form without written permission by copyright owner. Copyright infringers may face civil and criminal liability
Total Marks : 200.00
Prelims Test 22 - GS Test 15
( Insta Prelims Test Series 2024 7.0 ) Mark Scored : 83.54

country and its socio-cultural diversity.


The Indian federal system is based on the ‘Canadian model’ and not on the ‘American
model’.
The ‘Canadian model’ differs fundamentally from the ‘American model’ in so far as it
establishes a very strong centre.
However, the term ‘federation’ has nowhere been used in the Constitution. Instead,
Article 1 of the Constitution describes India as a ‘Union of States’.

36. Consider the following statements:


1. In India the power to initiate an amendment to the Constitution lies only with the Centre.
2. The Indian Federation is referred to as an indestructible Union of indestructible states.
3. There is a double system of courts in India where the federal laws are enforced by the federal
judiciary.
7
How many of the statements given above are correct? 277
6 3
A. Only one
560
B. Only two
- 9
C. All three m
D. None il.co
m a
4 ;g
Your Answer : A 6
Correct Answer : A
5 &#
va9
ta
as
Answer Justification :
i v
Statement 1 is correct: In India, sr
ti the power to initiate an amendment to the Constitution lies
r u
h the states can also propose an amendment to the Constitution.
only with the Centre. In US,
s
-
ta va
Unlike in other federations, the states in India have no right to territorial integrity. The
Parliamentivcan
s
a by unilateral action change the area, boundaries or name of any state.
i Sr
t
ru
Statement 2 is not correct: Moreover, it requires only a simple majority and not a special
Shmajority. Hence, the Indian Federation is “an indestructible Union of destructible states”.
The Indian Constitution has established an integrated judicial system with the Supreme Court at
the top and the state high courts below it.

Statement 3 is not correct: This single system of courts enforces both the Central laws as
well as the state laws. In US, on the other hand, there is a double system of courts whereby the
federal laws are enforced by the federal judiciary and the state laws by the state judiciary.

37. Consider the following statements:


1. The all-India services common to both Centre and the states uphold the principle of federalism
under the Constitution.
2. The Comptroller and Auditor-General of India audits the accounts of state governments and

instacourses.insightsonindia.com 24
© Insights Active Learning | All rights reserved - 18107. You may not reproduce, distribute or exploit the contents in any
form without written permission by copyright owner. Copyright infringers may face civil and criminal liability
Total Marks : 200.00
Prelims Test 22 - GS Test 15
( Insta Prelims Test Series 2024 7.0 ) Mark Scored : 83.54

hence is appointed after consulting the states.


3. The legislative competence of the Parliament in India cannot be extended without amending the
Constitution.

How many of the statements given above are not correct?


A. Only one
B. Only two
C. All three
D. None

Your Answer : B
Correct Answer : C

Answer Justification :
7 77
Statement 1 is not correct: There are all-India services (IAS, IPS, and IFS) which are
3 2common
to both the Centre and the states. The members of these services are recruited 6 06trained by
and
5
- 9 violate the
the Centre which also possess ultimate control over them. Thus, these services
principle of federalism under the Constitution. m
i l .co
Statement 2 is not correct: The Comptroller and Auditor-General m a of India audits the accounts
;g
of not only the Central government but also those of the4states. But, his appointment and
removal is done by the president without consulting # 6
the states.
&
a95
Statement 3 is not correct: Even in the v
talimited sphere of authority allotted to them, the
a s
states do not have exclusive control.vThe Parliament is empowered to legislate on any subject
i
isr a resolution to that effect in the national interest.
of the State List if Rajya Sabhatpasses
u
shr
a -
This means that the legislative competence of the Parliament can be extended without
ta v
amending the Constitution. Notably, this can be done when there is no emergency of any kind.
a s
Sriv
i
r ut
h
38.SConsider the following statements:
1. The Parliament can make laws for the whole or any part of the territory of India where the
territory includes the states and the union territories.
2. The Indian President can make regulations for good governance of certain Union Territories
which have the same force and effect as an act of Parliament.

Which of the statements given above is/are correct?


A. 1 only
B. 2 only
C. Both 1 and 2
D. Neither 1 nor 2

Your Answer : B
Correct Answer : C

instacourses.insightsonindia.com 25
© Insights Active Learning | All rights reserved - 18107. You may not reproduce, distribute or exploit the contents in any
form without written permission by copyright owner. Copyright infringers may face civil and criminal liability
Total Marks : 200.00
Prelims Test 22 - GS Test 15
( Insta Prelims Test Series 2024 7.0 ) Mark Scored : 83.54

Answer Justification :

Statement 1 is correct: The Parliament can make laws for the whole or any part of the
territory of India. The territory of India includes the states, the union territories, and any other
area for the time being included in the territory of India.

Statement 2 is correct: The President can make regulations for the peace, progress and good
government of the four Union Territories—the Andaman and Nicobar Islands, Lakshadweep,
Dadra and Nagar Haveli and Daman and Diu.

A regulation so made has the same force and effect as an act of Parliament. It may also repeal
or amend any act of Parliament in relation to these union territories.

The governor is empowered to direct that an act of Parliament does not apply to a scheduled
area in the state or apply with specified modifications and exceptions.
7
277
6 3
560
39. Consider the following statements:
- 9
1. Article 275 of the Indian Constitution empowers the Parliament to make omgrants to the states
l .c
which are in need of financial assistance.
m ai
2. The 80th Amendment of 2000 introduced changes in the scheme
4 ;g of the distribution of tax
revenues between the centre and the states. 6 #
5&
a9
Which of the statements given above is/are vcorrect?
A. 1 only
asta
B. 2 only
sr iv
i
C. Both 1 and 2
r ut
D. Neither 1 nor 2 sh
v a-
Your Answer a Ct
:s
a
riv : C
Correct Answer
S
i
r ut
ShAnswer Justification :
Statement 1 is correct: Article 275 empowers the Parliament to make grants to the states
which are in need of financial assistance and not to every state. Also, different sums may be
fixed for different states. These sums are charged on the Consolidated Fund of India every year.

The 80th Amendment of 2000 and the 88th Amendment of 2003 have introduced major changes
in the scheme of the distribution of tax revenues between the centre and the states.

Statement 2 is correct: The 80th Amendment was enacted to give effect to the
recommendations of the 10th Finance Commission.

40. Consider the following statements:


1. Unlike other federal Constitutions, the Indian Constitution does not contain the rule of ‘immunity
instacourses.insightsonindia.com 26
© Insights Active Learning | All rights reserved - 18107. You may not reproduce, distribute or exploit the contents in any
form without written permission by copyright owner. Copyright infringers may face civil and criminal liability
Total Marks : 200.00
Prelims Test 22 - GS Test 15
( Insta Prelims Test Series 2024 7.0 ) Mark Scored : 83.54

from mutual taxation’.


2. Article 263 of the Indian Constitution covers the establishment of an Inter-State Council to effect
coordination between the states.

Which of the statements given above is/are correct?


A. 1 only
B. 2 only
C. Both 1 and 2
D. Neither 1 nor 2

Your Answer : B
Correct Answer : B

Answer Justification :
77
27 also
Statement 1 is not correct: Like any other federal Constitution, the Indian Constitution
3
contain the rule of ‘immunity from mutual taxation’. 06
9 56
The Inter-State Water Disputes Act empowers the Central government m -
to set up an ad hoc
c o
a i l. relation to the waters of
tribunal for the adjudication of a dispute between two or more states in
an inter-state river or river valley. m
4 ;g
Statement 2 is correct: Article 263 contemplates the
& #6establishment of an Inter-State Council
5
a9
to effect coordination between the states and between Centre and states.
ta v
Thus, the President can establish suchaascouncil if at any time it appears to him that the public
sr iv
i
interest would be served by its establishment.
r ut
- sh
t a va
i v as
41. Consider the following statements regarding the proclamation of Emergency in India:

Sr approved by both the Houses of Parliament within one month from the date of its
1. It must be
i
t
issue.
h ru state is freed from the restrictions imposed by Article 19 while the emergency proclamation
S
2. The
is in operation.

Which of the statements given above is/are correct?


A. 1 only
B. 2 only
C. Both 1 and 2
D. Neither 1 nor 2

Your Answer : C
Correct Answer : C

Answer Justification :

Statement 1 is correct: The proclamation of Emergency must be approved by both the

instacourses.insightsonindia.com 27
© Insights Active Learning | All rights reserved - 18107. You may not reproduce, distribute or exploit the contents in any
form without written permission by copyright owner. Copyright infringers may face civil and criminal liability
Total Marks : 200.00
Prelims Test 22 - GS Test 15
( Insta Prelims Test Series 2024 7.0 ) Mark Scored : 83.54

Houses of Parliament within one month from the date of its issue.

Originally, the period allowed for approval by the Parliament was two months, but was reduced
by the 44th Amendment Act of 1978.

Statement 2 is correct: While a proclamation of national emergency is in operation, the state


is freed from the restrictions imposed by Article 19.

In other words, the state can make any law or can take any executive action abridging or taking
away the six Fundamental Rights guaranteed by Article 19.

42. Consider the following statements


1. The Constitution of India has adopted the system of proportional representation in the case of
7
Lok Sabha.
277
3
2. Under Proportional representation, every member of the legislature represents a geographical
area known as a constituency. 6 06
5
-9
3. The system of proportional representation aims to remove the defects of territorial
representation. om
ail.c
How many of the statements given above is/are correct? ;g m
A. Only one # 64
B. Only two
9 5&
a
C. All three
s tav
D. None a iv
sr
u ti
hr
Your Answer : A
Correct Answer : A - s
t a va
i v as
Answer Justification :
r
t
Thoughi Sthe Constitution has adopted the system of proportional representation in the case of
ru
Shrajya Sabha, it has not preferred the same system in the case of Lok Sabha.
Statement 1 is not correct: Instead, it has adopted the system of territorial representation
for the election of members to the Lok Sabha.

Statement 2 is not correct: Under territorial representation, every member of the legislature
represents a geographical area known as a constituency. From each constituency, only one
representative is elected.

Hence such a constituency is known as single-member constituency. In this system, a candidate


who secures majority of votes is declared elected. This simple majority system of
representation does not represent the whole electorate.

Statement 3 is correct: In other words, it does not secure due representation to minorities
(small groups). The system of proportional representation aims at removing the defects of
territorial representation.
instacourses.insightsonindia.com 28
© Insights Active Learning | All rights reserved - 18107. You may not reproduce, distribute or exploit the contents in any
form without written permission by copyright owner. Copyright infringers may face civil and criminal liability
Total Marks : 200.00
Prelims Test 22 - GS Test 15
( Insta Prelims Test Series 2024 7.0 ) Mark Scored : 83.54

43. Consider the following statements:


The Indian Constitution-
1. Fixed the term of Rajya Sabha members and empowered the President to curtail the term of
members chosen in the first Rajya Sabha.
2. Authorised the President to make provisions to govern the order of retirement of the members
of the Rajya Sabha.

Which of the statements given above is/are correct?


A. 1 only
B. 2 only
C. Both 1 and 2
D. Neither 1 nor 2
7
Your Answer : D 277
6 3
Correct Answer : D
560
- 9
Answer Justification : m
il.co
Statement 1 is not correct: The Constitution has not fixedm a term of office of members of
the
the Rajya Sabha and left it to the Parliament. 4;g
& #6
Accordingly, the Parliament in the Representation
a95 of the People Act (1951) provided that the
v
sta
term of office of a member of the Rajya Sabha shall be six years.
a
iv of India to curtail the term of members chosen in the
The act also empowered the president
i sr
first Rajya Sabha. r ut
- sh
a decided by lottery as to who should retire.
In the first batch, itvwas
t a
i v as
Statement
i Srthe2 order
is not correct: Further, the act also authorised the President to make provisions

r ut
to govern of retirement of the members of the Rajya Sabha.
Sh

44. Consider the following statements regarding the Indian Parliament:


1. The President is authorised to dissolve the Lok Sabha before the completion of five years which
cannot be challenged in a court of law.
2. The term of the Lok Sabha can be extended during the national emergency by a law of
Parliament only for six months at a time.

Which of the statements given above is/are correct?


A. 1 only
B. 2 only
C. Both 1 and 2
D. Neither 1 nor 2

instacourses.insightsonindia.com 29
© Insights Active Learning | All rights reserved - 18107. You may not reproduce, distribute or exploit the contents in any
form without written permission by copyright owner. Copyright infringers may face civil and criminal liability
Total Marks : 200.00
Prelims Test 22 - GS Test 15
( Insta Prelims Test Series 2024 7.0 ) Mark Scored : 83.54

Your Answer : D
Correct Answer : A

Answer Justification :

Unlike the Rajya Sabha, the Lok Sabha is not a continuing chamber. Its normal term is five years
from the date of its first meeting after the general elections, after which it automatically
dissolves.

Statement 1 is correct: However, the President is authorised to dissolve the Lok Sabha at any
time even before the completion of five years and this cannot be challenged in a court of law.

Statement 2 is not correct: Further, the term of the Lok Sabha can be extended during the
period of national emergency be a law of Parliament for one year at a time for any length of
time.
7
277
63
However, this extension cannot continue beyond a period of six months after the emergency
0
has ceased to operate. 5 6
-9
.c om
l
ai of the Parliament under
45. Consider the following statements regarding the membership
;g m
the Representation of People Act (1951): 4
1. If a person is elected to both Houses of Parliament,&he#6must intimate within ten days in which
5
House he desires to serve.
va9
ta to the other House, his seat in the first House
2. If a sitting member of one House is alsoselected
iva
becomes vacant.
i sr
3. A person cannot be a member
r utof both the Parliament and the state legislature at the same
time.
- sh
t a va
How many of the
i v as statements given above is/are correct?
Sr
A. Only one
B.tiOnly two
ru
Sh C. All three
D. None

Your Answer : C
Correct Answer : C

Answer Justification :

Option (c) is correct: A person cannot be a member of both Houses of Parliament at the same
time. Thus, the Representation of People Act (1951) provides for the following:

If a person is elected to both the Houses of Parliament, he must intimate within 10 days in
which House he desires to serve. In default of such intimation, his seat in the Rajya Sabha
becomes vacant.
If a sitting member of one House is also elected to the other House, his seat in the first

instacourses.insightsonindia.com 30
© Insights Active Learning | All rights reserved - 18107. You may not reproduce, distribute or exploit the contents in any
form without written permission by copyright owner. Copyright infringers may face civil and criminal liability
Total Marks : 200.00
Prelims Test 22 - GS Test 15
( Insta Prelims Test Series 2024 7.0 ) Mark Scored : 83.54

House becomes vacant.


If a person is elected to two seats in a House, he should exercise his option for one.
Otherwise, both seats become vacant.
Similarly, a person cannot be a member of both the Parliament and the state legislature
at the same time.
If a person is so elected, his seat in Parliament becomes vacant if he does not resign his
seat in the state legislature within 14 days

46. Consider the following statements regarding the Speaker of the Lok Sabha:
1. The Speaker derives his powers entirely from the Constitution of India.
2. He can be removed only by a resolution passed by the Lok Sabha by an absolute majority.
3. His work and conduct cannot be discussed and criticized in the Lok Sabha under any
circumstance.
7
How many of the statements given above is/are correct? 277
6 3
A. Only one
560
B. Only two - 9
C. All three m
D. None il.co
m a
4 ;g
Your Answer : B 6
Correct Answer : A
5 &#
va9
ta
Answer Justification :
i vas
Statement 1 is not correct: tThei sr
r u Speaker of the Lok Sabha derives his powers and duties
h the Constitution of India, the Rules of Procedure and Conduct of
from three sources, thatsis,
-
unspecified in s
va and Parliamentary Conventions (residuary powers that are unwritten or
Business of Lok Sabha,
taRules).
the
r iva
S
u ti
His powers of regulating procedure or conducting business or maintaining order in the House
r not subject to the jurisdiction of any Court.
are
Sh
Statement 2 is correct: He is provided with a security of tenure. He can be removed only by a
resolution passed by the Lok Sabha by an absolute majority (ie, a majority of the total members
of the House) and not by an ordinary majority (ie, a majority of the members present and voting
in the House).

Statement 3 is not correct: He cannot vote in the first instance. He can only exercise a
casting vote in the event of a tie. This makes the position of Speaker impartial.

His work and conduct cannot be discussed and criticised in the Lok Sabha except on a
substantive motion.

47. Consider the following statements regarding the Speaker Pro Tem:

instacourses.insightsonindia.com 31
© Insights Active Learning | All rights reserved - 18107. You may not reproduce, distribute or exploit the contents in any
form without written permission by copyright owner. Copyright infringers may face civil and criminal liability
Total Marks : 200.00
Prelims Test 22 - GS Test 15
( Insta Prelims Test Series 2024 7.0 ) Mark Scored : 83.54

1. The President administers oath to the Speaker Pro Tem of the Lok Sabha.
2. He presides over the first sitting of the newly-elected Lok Sabha.
3. It is a temporary office and hence does not have all the powers of the Speaker.

How many of the statements given above is/are correct?


A. Only one
B. Only two
C. All three
D. None

Your Answer : B
Correct Answer : B

Answer Justification :
77
27
Option (b) is correct: As provided by the Constitution, the Speaker of the last Lok Sabha
3
06
vacates his office immediately before the first meeting of the newly-elected Lok Sabha.
9 56
The President appoints a member of the Lok Sabha as the Speaker -
omPro Tem.
Usually, the seniormost member is selected for this.
il.c
The President himself administers oath to the Speaker mProaTem.
4;gLok Sabha.
He presides over the first sitting of the newly-elected
& #6
The Speaker Pro Tem has all the powers of the9 5
Speaker.
va
asta
sr iv
i
ut
48. Consider the following statements:
r
-
1. The Chairman of the Rajya shSabha can be removed from his office only if he is removed from the
t a va
office of the Vice-President.

i v as the Deputy Chairman of Rajya Sabha while presiding over the House, cannot
2. Like the Chairman,
vote in S r first instance.
the
t i
h ru
S Which of the statements given above is/are correct?
A. 1 only
B. 2 only
C. Both 1 and 2
D. Neither 1 nor 2

Your Answer : D
Correct Answer : C

Answer Justification :

Option (c) Is correct: The Chairman of the Rajya Sabha can be removed from his office only if
he is removed from the office of the Vice-President.

As a presiding officer, the powers and functions of the Chairman in the Rajya Sabha are similar

instacourses.insightsonindia.com 32
© Insights Active Learning | All rights reserved - 18107. You may not reproduce, distribute or exploit the contents in any
form without written permission by copyright owner. Copyright infringers may face civil and criminal liability
Total Marks : 200.00
Prelims Test 22 - GS Test 15
( Insta Prelims Test Series 2024 7.0 ) Mark Scored : 83.54

to those of the Speaker in the Lok Sabha.

Like the Chairman, the Deputy Chairman, while presiding over the House, cannot vote in the
first instance; he can only exercise a casting vote in the case of a tie.

Further, when a resolution for the removal of the Deputy Chairman is under consideration of
the House, he cannot preside over a sitting of the House, though he may be present.

49. In a Parliamentary Political System, which one of the following can be rightly
described as the “Alternative Prime Minister”?

A. The President
B. The Prime Minister
7
C. The Leader of the House
277
D. The Leader of the Opposition 6 3
560
Your Answer : D - 9
m
Correct Answer : D
il.co
m a
Answer Justification :
4 ;g
6
&#
5 has a unique institution called the ‘Shadow
Option (d) is correct: The British political system
a9
Cabinet’. v
a sta
It is formed by the Opposition r v to balance the ruling cabinet and to prepare its
iparty
s
i offices.
ut
members for future ministerial
r
sh almost every member in the ruling cabinet is ‘shadowed’ by a
In this shadow cabinet,
-
corresponding
t a vamember in the opposition cabinet.
i v as cabinet serves as the ‘alternate cabinet’ if there is change of government.
This shadow
Sr is whyHeIvor
That
iMinister’.
Jennings described the leader of Opposition as the ‘alternative Prime

r ut enjoys the status of a minister and is paid by the government.


Sh
50. Consider the following statements:
1. The office of the leader of the House is mentioned in the Constitution of India.
2. The office of ‘whip’ is not mentioned in the Constitution of India.
3. Only the ruling political party can have its own whip in the Parliament.

Which of the statements given above is/are correct?


A. 1 only
B. 2 only
C. 2 and 3 only
D. 1, 2 and 3

Your Answer : B
Correct Answer : B

instacourses.insightsonindia.com 33
© Insights Active Learning | All rights reserved - 18107. You may not reproduce, distribute or exploit the contents in any
form without written permission by copyright owner. Copyright infringers may face civil and criminal liability
Total Marks : 200.00
Prelims Test 22 - GS Test 15
( Insta Prelims Test Series 2024 7.0 ) Mark Scored : 83.54

Answer Justification :

Statement 1 is not correct: Though the offices of the leader of the House and the leader of
the Opposition are not mentioned in the Constitution of India, they are mentioned in the Rules
of the House and Parliamentary Statute respectively.

Statement 2 is correct: Office of ‘whip’, on the other hand, is mentioned neither in the
Constitution of India nor in the Rules of the House nor in a Parliamentary Statute. It is based on
the conventions of the parliamentary government.

Statement 3 is not correct: Every political party, whether ruling or Opposition has its own
whip in the Parliament. He is appointed by the political party to serve as an assistant floor
leader.

7
51. Consider the following statements regarding the Sessions of the Parliament:32
77
6
1. The power of adjournment sine die lies with the presiding officer of the House. 60
5
2. The Power of prorogation lies with the President of India. -9
3. The Prorogation brings to an end all the bills pending before the house.
.c om
ai l
How many of the statements given above is/are correct? ;g m
A. Only one # 64
&
B. Only two
a95
C. All three
s tav
D. None
r i va
u tis
Your Answer : A
shr
a-
Correct Answer : B
v
a sta :
Answer Justification
S riv
i
r ut
Adjournment sine die means terminating a sitting of Parliament for an indefinite period.
S h
In other words, when the House is adjourned without naming a day for reassembly, it is called
adjournment sine die.

Statement 1 is correct: The power of adjournment as well as adjournment sine die lies with
the presiding officer of the House.

The presiding officer (Speaker or Chairman) declares the House adjourned sine die, when the
business of a session is completed.

Statement 2 is correct: Within the next few days, the President issues a notification for
prorogation of the session. However, the President can also prorogue the House while in
session.

Statement 3 is not correct: Prorogation does not affect the bills or any other business
pending before the House. However, all pending notices (other than those for introducing bills)

instacourses.insightsonindia.com 34
© Insights Active Learning | All rights reserved - 18107. You may not reproduce, distribute or exploit the contents in any
form without written permission by copyright owner. Copyright infringers may face civil and criminal liability
Total Marks : 200.00
Prelims Test 22 - GS Test 15
( Insta Prelims Test Series 2024 7.0 ) Mark Scored : 83.54

lapse on prorogation and fresh notices have to be given for the next session.

In Britain, prorogation brings to an end all bills or any other business pending before the House.

52. Consider the following statements regarding the “Question Hour” of the Indian
Parliament:
1. It is an informal device available to the members of the Parliament to raise matters without any
prior notice.
2. It is an Indian innovation in the field of parliamentary procedures mentioned in the Rules of
Procedure.

Which of the statements given above is/are correct?


A. 1 only
7
B. 2 only
277
C. Both 1 and 2 6 3
D. Neither 1 nor 2 560
- 9
m
Your Answer : D
il.co
Correct Answer : A
m a
4 ;g
6
Answer Justification :
&#
95
a the zero hour is not mentioned in the
Option (a) is correct: Unlike the question
ta vhour,
i vas
Rules of Procedure. Thus it is an informal device available to the members of the
r
is any prior notice.
Parliament to raise matters without
t
h ru
a -s
The zero hour starts immediately after the question hour and lasts until the agenda for the day
(ie, regular business
ta v of the House) is taken up.
s
In other r iva the time gap between the question hour and the agenda is known as zero hour.
words,
i S
utan Indian innovation in the field of parliamentary procedures and has been in existence
It is
r
Shsince 1962.

53. With reference to the Point of Order, consider the following statements:
1. It suspends the proceedings before the House.
2. No debate is allowed on a point of order.
3. It is exclusively raised by a cabinet minister.

How many of the statements given above are correct?


A. Only one
B. Only two
C. All three
D. None

instacourses.insightsonindia.com 35
© Insights Active Learning | All rights reserved - 18107. You may not reproduce, distribute or exploit the contents in any
form without written permission by copyright owner. Copyright infringers may face civil and criminal liability
Total Marks : 200.00
Prelims Test 22 - GS Test 15
( Insta Prelims Test Series 2024 7.0 ) Mark Scored : 83.54

Your Answer : B
Correct Answer : B

Answer Justification :

Option (b) is correct: A member can raise a point of order when the proceedings of the House
do not follow the normal rules of procedure.

A point of order should relate to the interpretation or enforcement of the Rules of the
House or such articles of the Constitution that regulate the business of the House and
should raise a question that is within the cognizance of the Speaker.
It is an extraordinary device as it suspends the proceedings before the House. No debate
is allowed on a point of order.
It is usually raised by an opposition member in order to control the government.
7
277
3
54. Consider the following statements regarding the Financial Bill (I):
6 06
95 Lok Sabha.
1. Unlike the Money bill, it can be introduced only in the Rajya Sabha and not in the
-
2. Unlike the Money bill, it does not require a prior recommendation of the President for its
m
introduction. .co il
m a
Which of the statements given above is/are correct? 4 ;g
A. 1 only
& #6
B. 2 only
a95
C. Both 1 and 2 ta v
a s
D. Neither 1 nor 2 iv
tisr
u
Your Answer : D
shr
Correct Answer : D -
t a va
i v as
Answer Justification :
Sr
i
r ut (d) is correct: A financial bill (I) is a bill that contains not only any or all the matters
Option
Shmentioned in Article 110, but also other matters of general legislation.
For instance, a bill that contains a borrowing clause, but does not exclusively deal with
borrowing.
In two respects, a financial bill (I) is similar to a money bill—(a) both of them can be
introduced only in the Lok Sabha and not in the Rajya Sabha, and (b) both of them can be
introduced only on the recommendation of the president.
In all other respects, a financial bill (I) is governed by the same legislative procedure
applicable to an ordinary bill.

55. Consider the following statements regarding the judicial powers of the Indian
Parliament:
1. It can impeach the President for the violation of the Constitution.
2. It can recommend the removal of the Chief justice of the Supreme Court.

instacourses.insightsonindia.com 36
© Insights Active Learning | All rights reserved - 18107. You may not reproduce, distribute or exploit the contents in any
form without written permission by copyright owner. Copyright infringers may face civil and criminal liability
Total Marks : 200.00
Prelims Test 22 - GS Test 15
( Insta Prelims Test Series 2024 7.0 ) Mark Scored : 83.54

3. It can punish its members for the breach of its privileges.

How many of the statements given above is/are correct?


A. Only one
B. Only two
C. All three
D. None

Your Answer : C
Correct Answer : C

Answer Justification :

Option (c) is correct: The judicial powers and functions of the Parliament include the
following: 7
277
3
It can impeach the President for the violation of the Constitution.
6 06
It can remove the Vice-President from his office.
- 95
It can recommend the removal of judges (including chief justice) m of the Supreme Court
c o
and the high courts, chief election commissioner, comptroller
a i l. auditor general to the
and
president. m
4;gof its privileges or its contempt.
It can punish its members or outsiders for the breach
& #6
5
a9
s tav
56. Consider the following statements regarding the Removal of a Supreme Court Judge:
a
1. The President can issue the removalivorder of the judge even if he has not received any address
tisr
by the Parliament for such removal.
u
hr regulates the procedure relating to the removal of a judge of the
2. The Judges Enquiry Act (1968)
s
Supreme Court by the -process of impeachment.
t a va
s
astatements
Which of the
r i v given above is/are correct?
S
A. i1 only
u t
h r B. 2 only
S C. Both 1 and 2
D. Neither 1 nor 2

Your Answer : B
Correct Answer : B

Answer Justification :

A judge of the Supreme Court can be removed from his Office by an order of the president.

Statement 1 is not correct: The President can issue the removal order only after an address
by Parliament has been presented to him in the same session for such removal.

The address must be supported by a special majority of each House of Parliament (ie, a majority
of the total membership of that House and a majority of not less than two-thirds of the

instacourses.insightsonindia.com 37
© Insights Active Learning | All rights reserved - 18107. You may not reproduce, distribute or exploit the contents in any
form without written permission by copyright owner. Copyright infringers may face civil and criminal liability
Total Marks : 200.00
Prelims Test 22 - GS Test 15
( Insta Prelims Test Series 2024 7.0 ) Mark Scored : 83.54

members of that House present and voting).

Statement 2 is correct: The Judges Enquiry Act (1968) regulates the procedure relating to the
removal of a judge of the Supreme Court by the process of impeachment:

A removal motion signed by 100 members (in the case of Lok Sabha) or 50 members (in
the case of Rajya Sabha) is to be given to the Speaker/Chairman.
The Speaker/Chairman may admit the motion or refuse to admit it

57. The Original Jurisdiction of the Supreme Court of India extends to-

A. The dispute between the Centre and the States.


B. The dispute arising out of any pre-Constitution treaty.
C. The adjustment of pensions between the Centre and the states. 7
D. The matters referred to the Finance Commission. 277
6 3
Your Answer : A 560
- 9
Correct Answer : A
m
il.co
Answer Justification :
m a
4 ;g
Option (a) is correct: As a federal court, the Supreme
& #6 Court having original jurisdiction
decides the disputes between different units of9the 5 Indian Federation. More elaborately, any
va
sta
dispute between:
a
iv or
the Centre and one or more
i srstates;
the Centre and any stater utor states on one side and one or more states on the other; or
h Supreme Court does not extend to the following:
sthe
-
This jurisdiction of

t a va out of any pre-Constitution treaty, agreement, covenant, engagement,


A dispute arising
sanad or
i v asother similar instrument.
r
ASdispute arising out of any treaty, agreement, etc., which specifically provides that the
t i
h ru said jurisdiction does not extent to such a dispute.
S Inter-state water disputes.
Matters referred to the Finance Commission.
Adjustment of certain expenses and pensions between the Centre and the states.

58. Which one of the following Articles of the Indian Constitution explicitly confers the
power of Judicial Review on the Supreme Court and the High Courts?

A. Article 13
B. Article 14
C. Article 15
D. Article 16

Your Answer : A
Correct Answer : A

instacourses.insightsonindia.com 38
© Insights Active Learning | All rights reserved - 18107. You may not reproduce, distribute or exploit the contents in any
form without written permission by copyright owner. Copyright infringers may face civil and criminal liability
Total Marks : 200.00
Prelims Test 22 - GS Test 15
( Insta Prelims Test Series 2024 7.0 ) Mark Scored : 83.54

Answer Justification :

Option (a) is correct: Though the phrase ‘Judicial Review’ has nowhere been used in the
Constitution, the provisions of several Articles explicitly confer the power of judicial review on
the Supreme Court and the High Courts.

These provisions are explained below:

Article 13 declares that all laws that are inconsistent with or in derogation of the
Fundamental Rights shall be null and void.
Article 32 guarantees the right to move the Supreme Court for the enforcement of the
Fundamental Rights and empowers the Supreme Court to issue directions or orders or
writs for that purpose.
Article 131 provides for the original jurisdiction of the Supreme Court in centre–state and
inter-state disputes.
7
77
Article 132 provides for the appellate jurisdiction of the Supreme Court in constitutional
2
cases. 63
560
- 9
m
.co
th
59. The 7 Constitutional Amendment Act of 1956-
ail
A. Changed the basis for appeals in Supreme Court of India
m
;g in case of Civil Suits.
4
B. #6 to raising of taxes.
Amended the Union List and State List with respect
&
C. 5
Extended the reservation of seats for the9Scheduled Castes in the Lok Sabha.
D. va
Facilitated the appointment of sameaperson as a governor for two or more states.
ast
ri v
Your Answer :
ti s
u
Correct Answer : D
shr
a -
tav :
Answer Justification
i v as
r is correct: The governor is the chief executive head of the state. But, like the
OptionS(d)
t i
h ru
president, he is a nominal executive head (titular or constitutional head).
S
The governor also acts as an agent of the central government. Therefore, the office of governor
has a dual role.

Usually, there is a governor for each state, but the 7th Constitutional Amendment Act of 1956
facilitated the appointment of the same person as a governor for two or more states.

60. Consider the following statements:


1. The Indian Constitution does not specify the size of the state council of ministers.
2. The term ‘ministry’ in India is used only in the centre and not in the states.
3. The Indian Constitution provides for the abolition of legislative councils in states.

How many of the statements given above are correct?

instacourses.insightsonindia.com 39
© Insights Active Learning | All rights reserved - 18107. You may not reproduce, distribute or exploit the contents in any
form without written permission by copyright owner. Copyright infringers may face civil and criminal liability
Total Marks : 200.00
Prelims Test 22 - GS Test 15
( Insta Prelims Test Series 2024 7.0 ) Mark Scored : 83.54

A. Only one
B. Only two
C. All three
D. None

Your Answer : A
Correct Answer : C

Answer Justification :

Statement 1 is correct: The Constitution does not specify the size of the state council of
ministers or the ranking of ministers.

They are determined by the chief minister according to the exigencies of the time and
requirements of the situation. 7
277
3
06 and not
Statement 2 is correct: The term ‘ministry’ or ‘ministries’ is used only in the centre
6
5and not
in the states. In other words, the state government is divided into departments
ministries. -9
.c om
Statement 3 is correct: The Constitution provides for the abolition
l
ai or creation of legislative
councils in states. ;g m
4
& #6
5 council (where it already exists) or create it
Accordingly, the Parliament can abolish a legislative
9
a
s tav
(where it does not exist), if the legislative assembly of the concerned state passes a resolution
to that effect.
r i va
u tis
shr
61. Consider the following
v a - statements :
Statement-I : sta
The form of a
ivnumerals to be used for the official purposes of the Union of India has to
Sr
be thei international form of Indian numerals and not the Devanagari form of
t
ru
numerals.
h
SStatement-II :
English language would continue to be used for all the official purposes of the Union
for which it was being used before 1950.

Which one of the following is correct in respect of the above statements?


A. Both Statement-I and Statement-II are correct and Statement-II is the correct explanation
for Statement-I
B. Both Statement-I and Statement-II are correct and Statement-II is not the correct
explanation for Statement-I
C. Statement-I is correct but Statement-II is incorrect
D. Statement-I is incorrect but Statement-II is correct

Your Answer : C
Correct Answer : B

instacourses.insightsonindia.com 40
© Insights Active Learning | All rights reserved - 18107. You may not reproduce, distribute or exploit the contents in any
form without written permission by copyright owner. Copyright infringers may face civil and criminal liability
Total Marks : 200.00
Prelims Test 22 - GS Test 15
( Insta Prelims Test Series 2024 7.0 ) Mark Scored : 83.54

Answer Justification :

The Constitution contains the following provisions in respect of the official language of the
Union.

1. Hindi written in Devanagari script is to be the official language of the Union. But, the form of
numerals to be used for the official purposes of the Union has to be the international
form of Indian numerals and not the Devanagari form of numerals.

Hence, statement I is correct.

7
277
3
06 (i.e., from
2. However, for a period of fifteen years from the commencement of the Constitution
6
1950 to 1965), the English language would continue to be used for all9the 5 official
purposes of the Union for which it was being used before 1950.m
-
i l .co
;g ma
4
#6 explanation of Statement I.
Hence, statement II is correct but it is not the correct
&
5
va9
a sta
62. Consider the following statements sr iv regarding Protection of Linguistic Minorities:
i
r ut in the state should provide adequate facilities for instruction in
1. Every state and a local authority
the mother-tongue at the- shprimary stage of education to children belonging to linguistic minority
groups.
t a va
2. The Parliament
i v asof India can issue necessary directions for this purpose.
Sr Minister of India appoints a special officer for linguistic minorities to investigate all
3. The Prime
i
t
h ru relating to the constitutional safeguards for linguistic minorities.
matters
S
How many of the above statements are incorrect?
A. Only one
B. Only two
C. All three
D. None

Your Answer : B
Correct Answer : B

Answer Justification :

Protection of Linguistic Minorities

instacourses.insightsonindia.com 41
© Insights Active Learning | All rights reserved - 18107. You may not reproduce, distribute or exploit the contents in any
form without written permission by copyright owner. Copyright infringers may face civil and criminal liability
Total Marks : 200.00
Prelims Test 22 - GS Test 15
( Insta Prelims Test Series 2024 7.0 ) Mark Scored : 83.54

Every state and a local authority in the state should provide adequate facilities for
instruction in the mother-tongue at the primary stage of education to children belonging
to linguistic minority groups.

Hence, statement 1 is correct.

The president can issue necessary directions for this purpose.

Hence, statement 2 is incorrect.


7
277
3
6 06
The president should appoint a special officer for linguistic minorities
- 95to investigate all
matters relating to the constitutional safeguards for linguistic minorities
m and to report to
c o
him.
a il.
4 ;gm
& #6
Hence, statement 3 is incorrect. 5
va9
asta
sr iv
i
ut all such reports before the Parliament and send to the state
The president should place
r
sh
government concerned.
-
t a va
s
63. Consider rtheivafollowing statements regarding Public Services:
i S Services Act of 1951 authorised the State Governments to make rules for the
1. The tAll-India
ru
hregulation of service conditions of the members of all-India services.
S
2. The members of these services are recruited and trained by the Central government and they
belong to central cadres.
3. The Central government obtains the services of these officers on deputation under the well-
known tenure system.
4. Any disciplinary action (imposition of penalties) against these officers can only be taken by the
Central government.

How many of the above statements are correct?


A. Only one
B. Only two
C. Only three
D. All four

Your Answer : B

instacourses.insightsonindia.com 42
© Insights Active Learning | All rights reserved - 18107. You may not reproduce, distribute or exploit the contents in any
form without written permission by copyright owner. Copyright infringers may face civil and criminal liability
Total Marks : 200.00
Prelims Test 22 - GS Test 15
( Insta Prelims Test Series 2024 7.0 ) Mark Scored : 83.54

Correct Answer : B

Answer Justification :

The All-India Services Act of 1951 authorised the Central government to make rules in
consultation with the state governments for the regulation of recruitment and service conditions
of the members of all-India services.

Hence, statement 1 is incorrect.

The members of these services are recruited and trained by the Central government but are
assigned to different states for work. They belong to different state cadres; the Centre
having no cadre of its own in this regard.

Hence, statement 2 is incorrect.


7
2 77
63 they
They serve the Central government on deputation and after completing their fixed tenure
0
56 of these
go back to their respective states. The Central government obtains the services
9
officers on deputation under the well-known tenure system. -
m
Hence, statement 3 is correct. i l .co
m a
g
4; the immediate control is vested in
The ultimate control lies with the Central government while
#6
the state governments. Any disciplinary action & (imposition of penalties) against these
9 5
officers can only be taken by the Centralagovernment.
s tav
Hence, statement 4 is correct. iva
tisr
u
s hr
v a - statements regarding the "Liability for Contracts"
sta
64. Consider the following
v
1. They must be aexpressed to be made by the president or governor, as the case may be.
S
2. They mustribe executed on behalf of the president or governor, as the case may be.
i
r ut must be executed by such person or in such manner as the president or governor may
3. They
Shdirect or authorise.
How many of the above conditions are mandatory in case of Union or State governments?
A. Only one
B. Only two
C. All three
D. None

Your Answer : A
Correct Answer : C

Answer Justification :

Liability for Contracts

instacourses.insightsonindia.com 43
© Insights Active Learning | All rights reserved - 18107. You may not reproduce, distribute or exploit the contents in any
form without written permission by copyright owner. Copyright infringers may face civil and criminal liability
Total Marks : 200.00
Prelims Test 22 - GS Test 15
( Insta Prelims Test Series 2024 7.0 ) Mark Scored : 83.54

Under the exercise of its executive power, the Union or a state can enter into contracts for the
acquisition, holding and disposal of property, or to carry on any trade or business, or for any
other purpose. But, the Constitution lays down three conditions which must be fulfilled by such
contracts:

(a) They must be expressed to be made by the president or governor, as the case may be;

(b) They must be executed on behalf of the president or governor, as the case may be; and

(c) They must be executed by such person or in such manner as the president or governor may
direct or authorise.

These conditions are mandatory and not merely directory in nature.

Hence, all these statements are correct.


7
277
6 3
560
65. Consider the following statements:
- 9
1. Only parliament, and not the state legislatures, has power to regulate the
om recruitment and the
conditions of service of the persons appointed to public services.il. c
2. The Parliament or the state legislature can impose ‘reasonable’ m arestrictions on the Fundamental
;g
64
Rights of public servants in the interests of integrity, efficiency,
#
devotion to duty and so on.

Which of the above statements is/are correct?a9


5&
A. 1 only s tav
B. 2 only r i va
C. Both 1 and 2 u tis
D. Neither 1 nor 2 sh
r
v a-
Your Answer : sBta
a
riv : B
Correct Answer
S
u ti
r
ShAnswer Justification :
Article 309 empowers the Parliament and the state legislatures to regulate the
recruitment and the conditions of service of the persons appointed to public services and posts
under the Centre and the states, respectively.

Hence, statement 1 is incorrect.

Under this provision, the Parliament or the state legislature can impose ‘reasonable’
restrictions on the Fundamental Rights of public servants in the interests of integrity,
honesty, efficiency, discipline, impartiality, secrecy, neutrality, anonymity, devotion to duty and
so on.

Hence, statement 2 is correct.

instacourses.insightsonindia.com 44
© Insights Active Learning | All rights reserved - 18107. You may not reproduce, distribute or exploit the contents in any
form without written permission by copyright owner. Copyright infringers may face civil and criminal liability
Total Marks : 200.00
Prelims Test 22 - GS Test 15
( Insta Prelims Test Series 2024 7.0 ) Mark Scored : 83.54

66. Consider the following statements :


Statement-I :
It depends upon the state's legislature to adopt any one or more of the languages in
use in the state.
Statement-II :
The choice of the state is limited to the languages enumerated in the Eighth Schedule
of the Indian Constitution.

Which one of the following is correct in respect of the above statements?


A. Both Statement-I and Statement-II are correct and Statement-II is the correct explanation
for Statement-I
B. Both Statement-I and Statement-II are correct and Statement-II is not the correct
explanation for Statement-I
C. Statement-I is correct but Statement-II is incorrect
7
D. Statement-I is incorrect but Statement-II is correct 77 2
3
06
Your Answer : C
9 56
Correct Answer : C -
.c om
Answer Justification : ail
4 ;gm
The Constitution does not specify the official language 6 of different states. In this regard, it
makes the following provisions: & #
a 95
The legislature of a state may adopt any s tavone or more of the languages in use in the
state or Hindi as the official language
r i vaof that state. Until that is done, English is to continue as
official language of that state. ti
s
u
shr
Hence, statementaI is - correct.
v
a sta of the state is not limited to the languages enumerated in the
riv
Notably, the choice
S
u ti
Eighth Schedule of the Constitution.
r
ShHence, statement II is incorrect.

67. Consider the following statements:


1. The Constitution does not specify the castes or tribes which are to be called the SCs or the STs.
2. The lists of the SCs or STs are common for states and union territories.
3. Any inclusion or exclusion of any caste or tribe from Presidential notification can be done only
by a subsequent Presidential notification.

How many of the above statements are correct?


A. Only one
B. Only two
C. All three
D. None

instacourses.insightsonindia.com 45
© Insights Active Learning | All rights reserved - 18107. You may not reproduce, distribute or exploit the contents in any
form without written permission by copyright owner. Copyright infringers may face civil and criminal liability
Total Marks : 200.00
Prelims Test 22 - GS Test 15
( Insta Prelims Test Series 2024 7.0 ) Mark Scored : 83.54

Your Answer : D
Correct Answer : A

Answer Justification :

The Constitution does not specify the castes or tribes which are to be called the SCs
or the STs.

Hence, statement 1 is correct.

It leaves to the President the power to specify as to what castes or tribes in each state and
union territory are to be treated as the SCs and STs. Thus, the lists of the SCs or STs vary
from state to state and union territory to union territory.

Hence, statement 2 is incorrect.


7
2 77
In case of the states, the President issues the notification after consulting the governor
0 63 of the
state concerned. But, any inclusion or exclusion of any caste or tribe from6Presidential
5
notification can be done only by the Parliament and not by a subsequent - 9 Presidential
notification. Presidents have issued several orders specifying the SCs
.c omand STs in different
states and union territories and these have also been amendedaby l
i the Parliament.
;g m
4
#6
Hence, statement 3 is incorrect.
5 &
v a9
s ta
68. Consider the following statements: iva
sr
u ti
1. The President may appoint a commission to investigate the conditions of socially and
r
sh
educationally backward classes.
-
v a
2. The report of the commission is to be placed before the Parliament, along with action taken
memorandum.sta
a
S riv
Which
u ti of the above statements is/are correct?
r A. 1 only
Sh B. 2 only
C. Both 1 and 2
D. Neither 1 nor 2

Your Answer : C
Correct Answer : C

Answer Justification :

The President may appoint a commission to investigate the conditions of socially and
educationally backward classes and to recommend the steps to improve their condition.

Hence, statement 1 is correct.

The report of the commission is to be placed before the Parliament, along with action taken

instacourses.insightsonindia.com 46
© Insights Active Learning | All rights reserved - 18107. You may not reproduce, distribute or exploit the contents in any
form without written permission by copyright owner. Copyright infringers may face civil and criminal liability
Total Marks : 200.00
Prelims Test 22 - GS Test 15
( Insta Prelims Test Series 2024 7.0 ) Mark Scored : 83.54

memorandum.

Hence, statement 2 is correct.

69. Consider the following statements regarding the classical languages:


1. Once a language is declared classical, it gets financial assistance for setting up a centre of
excellence for the study of that language.
2. The University Grants Commission can be requested to create a certain number of professional
chairs for classical languages.

Which of the above statements is/are incorrect?


A. 1 only
B. 2 only 7
C. Both 1 and 2 277
D. Neither 1 nor 2 6 3
560
- 9
Your Answer : D
m
Correct Answer : D
il.co
m a
Answer Justification : 4 ;g
6
5 &#
a9
Benefits of classical languages:
ta v
Once a language is declared classical,aitsgets financial assistance for setting up a centre of
r iv
excellence for the study of that slanguage and also opens up an avenue for two major awards
ti
ru the University Grants Commission can be requested to
for scholars of eminence. Besides,
h
s
- least in Central Universities - a certain number of professional chairs for
create - to begin with at
classical languagesv a for scholars of eminence in the language.
a st
iv a
r the statements are correct.
Hence, both
S
i
r ut
Sh
70. Consider the following statements :
Statement-I :
Anti-Defection law has brought transparency into the Indian Political System.
Statement-II :
It specifies the circumstances under which changing of political parties by legislators
invites action under the law.

Which one of the following is correct in respect of the above statements?


A. Both Statement-I and Statement-II are correct and Statement-II is the correct explanation
for Statement-I
B. Both Statement-I and Statement-II are correct and Statement-II is not the correct
explanation for Statement-I
C. Statement-I is correct but Statement-II is incorrect
D. Statement-I is incorrect but Statement-II is correct

instacourses.insightsonindia.com 47
© Insights Active Learning | All rights reserved - 18107. You may not reproduce, distribute or exploit the contents in any
form without written permission by copyright owner. Copyright infringers may face civil and criminal liability
Total Marks : 200.00
Prelims Test 22 - GS Test 15
( Insta Prelims Test Series 2024 7.0 ) Mark Scored : 83.54

Your Answer : A
Correct Answer : A

Answer Justification :

Anti-defection law

It is covered under the Tenth Schedule of the Indian Constitution to promote transparency into
the Indian Political System.

Hence, statement I is correct.


7
277
3
6 06
5 legislators
It specifies the circumstances under which changing of political parties9by
-
invites action under the law.
.c om
ai l
;g m
4
Hence, statement II is correct and it is the correct
& #6 explanation of Statement I.
5
va9
a sta
sr
It was added to the Constitutioniv by the 52nd Amendment Act, 1985.
ti an independent MLA, too, joins a party after the election.
uwhich
r
It includes situations in
- sh
t a va
i v as
71. Consider the following statements regarding the facilities provided to the National

Sr 'National' political party requires only five proposers to file nomination.


Political Party:
t i
1. Recognized
ru National parties receive land or building from the government to establish their party office.
2. hThe
S
3. National parties can have upto 40 star campaigners while other can have upto 20 star
campaigners.

How many of the above statements are correct?


A. Only one
B. Only two
C. All three
D. None

Your Answer : D
Correct Answer : B

Answer Justification :

instacourses.insightsonindia.com 48
© Insights Active Learning | All rights reserved - 18107. You may not reproduce, distribute or exploit the contents in any
form without written permission by copyright owner. Copyright infringers may face civil and criminal liability
Total Marks : 200.00
Prelims Test 22 - GS Test 15
( Insta Prelims Test Series 2024 7.0 ) Mark Scored : 83.54

Facilities to National Party:

Recognition as a national or a State party ensures that the election symbol of that party
can't be used by any other political party in polls across India.
Recognized 'State' and 'National' party requires only one proposer to file nomination.

Hence, statement 1 is incorrect.

The National parties receive land or building from the government to establish their
party office.

Hence, statement 2 is correct.

The recognized 'state' and National Parties are given two sets of electoral rolls free of cost
by the Election Commission. In addition to this the candidates contesting on the ticket of
National or state party get a copy of the electoral roll free of cost during the general77
elections. 3 27
6
National Parties get the time slot on the national and state television & Radio 06to address
5
the people and convey their message to the mass people. -9
National parties can have upto 40 star campaigners while other
.c om can have upto 20
star campaigners. The expenditure incurred on the travelling l
ai and other expenses of
g m
star campaigners is not included in the election expenditure of the party candidate.
;
#64
Hence, statement 3 is correct.
5 &
va9
ta
i vas
72. Consider the followings: ti sr
r u
sh all eligible voters
1. Introduction of Voter IDs for
-
a of liquor/money during elections
2. Prohibition of distribution
v
3. Prohibition on s ta of official machinery for campaigning
use
4. Autonomous a
iv status to Election Commission of India
i Sr
t
ru many of the above were the reforms started by T.N. Seshan in Electoral System in India?
Sh A. Only one
How

B. Only two
C. Only three
D. All four

Your Answer : C
Correct Answer : D

Answer Justification :

Major Election Reforms started by the T.N. Seshan were:

1. Introduction of Voter IDs for all eligible voters.

2. Strict actions for implementation Election Code of Conduct

instacourses.insightsonindia.com 49
© Insights Active Learning | All rights reserved - 18107. You may not reproduce, distribute or exploit the contents in any
form without written permission by copyright owner. Copyright infringers may face civil and criminal liability
Total Marks : 200.00
Prelims Test 22 - GS Test 15
( Insta Prelims Test Series 2024 7.0 ) Mark Scored : 83.54

3. Set expenditure limit for candidates in the elections

4. Prohibition of distribution of liquor/money during elections.

5. Prohibited bribing or intimidating voters.

6. Prohibition on use of official machinery for campaigning.

7. Implementation of the law in election process.

8. Autonomous status to Election Commission of India.

9. Prohibition of use of loudspeakers and high volume music without prior written permission.

10. Prohibited candidates from demanding votes on the basis of caste or communal feelings.
7
11. Prohibition of use of religious place for election campaigns. 277
6 3
Hence, all four statements are correct. 560
- 9
m
il.co
m a
4 ;g
6
5 &#
va9
s ta
73. Consider the following statements:
r i va
#18368
u tis
shr
1. VVPAT Machines were introduced to increase transparency and eliminate doubts about the
-
va
accuracy of the EVMs.
2. VVPAT machinesta can be accessed by polling officers only.
s
va are separate entities and are not connected to any network.
3. EVMs and iVVPATs
i Sr
t
ru many of the above statements are incorrect?
How
Sh A. Only one
B. Only two
C. All three
D. None

Your Answer : B
Correct Answer : D

Answer Justification :
VVPAT is a verification printer machine attached to electronic voting machines (EVMs)
that allows voters to verify that their votes have been recorded accurately.
VVPAT Machines were first introduced in India in the 2014 Lok Sabha elections and were
introduced to increase transparency and eliminate doubts about the accuracy of the
EVMs.

instacourses.insightsonindia.com 50
© Insights Active Learning | All rights reserved - 18107. You may not reproduce, distribute or exploit the contents in any
form without written permission by copyright owner. Copyright infringers may face civil and criminal liability
Total Marks : 200.00
Prelims Test 22 - GS Test 15
( Insta Prelims Test Series 2024 7.0 ) Mark Scored : 83.54

Hence statements 1 is correct.

VVPAT machines can be accessed by polling officers only.

Hence, statements 2 is correct.

According to ECI, EVMs and VVPATs are separate entities and are not connected to
any network.

Hence, statements 3 is correct.

74. Consider the following provisions relating to the Representation of the People Act,
1950:
1. It lays procedure for the preparation of electoral rolls and the manner of filling seats.
7 77
2
63
2. The 1950 Act permits the registration of persons in electoral rolls who are ordinarily resident in
0
56 in a
a constituency.
3. According to this Act, Returning Officer is responsible for the conduct of the- 9
election
constituency and returns an elected candidate. o m
ail.c
How many of the above statements are correct?
;g m
4
#6
A. Only one
B. Only two
5 &
C. All three v a9
D. None sta
r i va
Your Answer : B u tis
Correct Answer : C shr
-
t a va
i v as
Answer Justification :

i Sr
r ut
Representation of the People Act (RPA), 1950: Key Provisions
Sh

Lays down procedures for delimitation of constituencies.


Provides for the allocation of seats in the House of the People and in the Legislative
Assemblies and Legislative Councils of States.
Lays procedure for the preparation of electoral rolls and the manner of filling seats.

Hence, statement 1 is correct.

Lays down the qualification of voters.


The 1950 Act permits the registration of persons in electoral rolls who are ordinarily

instacourses.insightsonindia.com 51
© Insights Active Learning | All rights reserved - 18107. You may not reproduce, distribute or exploit the contents in any
form without written permission by copyright owner. Copyright infringers may face civil and criminal liability
Total Marks : 200.00
Prelims Test 22 - GS Test 15
( Insta Prelims Test Series 2024 7.0 ) Mark Scored : 83.54

resident in a constituency.

Hence, statement 2 is correct.

According to this Act, Returning Officer is responsible for the conduct of the election
in a constituency and returns an elected candidate.

Hence, statement 3 is correct.

7
277
3
75. Consider the following statements :
6 06
Statement-I :
95
The constitution has specified the classes of citizens who are to be -called the socially
and educationally backward classes.
.c om
Statement-II : ai l
;g
The 102nd Amendment Act of 2018 empowered the President m to specify the socially
and educationally backward classes in relation to 6a4state or union territory.
5 &#
Which one of the following is correct in respectva9of the above statements?
a
stare correct and Statement-II is the correct explanation
A. Both Statement-I and Statement-II
va
for Statement-I
i sri
ut
B. Both Statement-I andrStatement-II are correct and Statement-II is not the correct
sh
explanation for Statement-I
-
acorrect
C. Statement-I is
t a v but Statement-II is incorrect
s
iva
D. Statement-I is incorrect but Statement-II is correct
r
t i S :D
ru
Your Answer
ShCorrect Answer : D
Answer Justification :

The constitution has not specified the classes of citizens who are to be called the socially and
educationally backward classes, also known as Other Backward Classes (OBCs).

Hence, statement I is incorrect.

The 102nd Amendment Act of 2018 empowered the President to specify the socially and
educationally backward classes in relation to a state or union territory.

Hence, statement II is correct.

instacourses.insightsonindia.com 52
© Insights Active Learning | All rights reserved - 18107. You may not reproduce, distribute or exploit the contents in any
form without written permission by copyright owner. Copyright infringers may face civil and criminal liability
Total Marks : 200.00
Prelims Test 22 - GS Test 15
( Insta Prelims Test Series 2024 7.0 ) Mark Scored : 83.54

76. Consider the following statements:


1. If a member, after being elected as the presiding officer of the House, voluntarily gives up the
membership of his party or rejoins it after he ceases to hold that office.
2. If a member goes out of his party as a result of a merger of the party with another party.

Which of the above is/are the exceptions to the Anti-defection Law?


A. 1 only
B. 2 only
C. Both 1 and 2
D. Neither 1 nor 2

Your Answer : C
Correct Answer : C
7
Answer Justification : 277
3
6 06
The above disqualification on the ground of defection does not apply in the9following5 two
cases: -
.c om
(a) If a member goes out of his party as a result of a merger of a the
l
i party with another party. A
merger takes place when two- thirds of the members of the
m
;gparty have agreed to such merger.
4
& #6
Hence, statement 1 is correct. 5
va9
(b) If a member, after being elected asathe stapresiding officer of the House, voluntarily gives up
the membership of his party or rejoins
s r iv it after he ceases to hold that office. This exemption has
been provided in view of the u ti and impartiality of this office.
dignity
r
- sh
Hence, statement
t a va2 is correct.
i v as
i Sr
t the following statements regarding Pressure Groups:
ru
77. Consider
h
S A pressure group is a group of people who are organised actively for promoting and defending
1.
their common interest.
2. The term ‘pressure group’ originated in the UK.
3. They are concerned with specific programmes and issues and their activities are confined to the
protection and promotion of the interests of their members by influencing the government.

How many of the above statements are correct?


A. Only one
B. Only two
C. All three
D. None

Your Answer : B
Correct Answer : B

instacourses.insightsonindia.com 53
© Insights Active Learning | All rights reserved - 18107. You may not reproduce, distribute or exploit the contents in any
form without written permission by copyright owner. Copyright infringers may face civil and criminal liability
Total Marks : 200.00
Prelims Test 22 - GS Test 15
( Insta Prelims Test Series 2024 7.0 ) Mark Scored : 83.54

Answer Justification :

The term ‘pressure group’ originated in the USA. A pressure group is a group of people who are
organised actively for promoting and defending their common interest. It is so called as it
attempts to bring a change in the public policy by exerting pressure on the government. It acts
as a liaison between the government and its members.

Hence, statement 2 is incorrect but statement 1 is correct.

The pressure groups are also called interest groups or vested groups. They are different from
the political parties in that they neither contest elections nor try to capture political power. They
are concerned with specific programmes and issues and their activities are confined to the
protection and promotion of the interests of their members by influencing the government.

Hence, statement 3 is correct.


7
277
6 3
560
78. Consider the following statements :
- 9
Statement-I : m
l .co
Section 126 of the Representation of the People Act, 1951 prohibits
i displaying any
election matter by mean of cinematograph, television or a
other similar apparatus,
;gmfor conclusion of poll.
during the period of 48 hours ending with the hour4fixed
Statement-II :
& #6
95 incurring of expenditure on, inter
Section 171H of the Indian Penal Code prohibits
vaof the contesting candidate.
alia, advertisement without the authority
a sta
sr ivin respect of the above statements?
i
Which one of the following is correct
ut
A. Both Statement-I andrStatement-II are correct and Statement-II is the correct explanation
for Statement-I - sh
a
B. Both Statement-I
t va and Statement-II are correct and Statement-II is not the correct
v as for Statement-I
explanation
i
i Sr
C. Statement-I is correct but Statement-II is incorrect
t
ruD. Statement-I is incorrect but Statement-II is correct
Sh
Your Answer : B
Correct Answer : B

Answer Justification :

Compliance to Election Laws:-

1. Section 126 of the Representation of the People Act, 1951: It prohibits displaying
any election matter by mean of cinematograph, television or other similar apparatus,
during the period of 48 hours ending with the hour fixed for conclusion of poll.

Hence, statement I is correct.

instacourses.insightsonindia.com 54
© Insights Active Learning | All rights reserved - 18107. You may not reproduce, distribute or exploit the contents in any
form without written permission by copyright owner. Copyright infringers may face civil and criminal liability
Total Marks : 200.00
Prelims Test 22 - GS Test 15
( Insta Prelims Test Series 2024 7.0 ) Mark Scored : 83.54

1. (ii) Section 126A of the Representation of the People Act, 1951: It prohibits
conduct of exit poll and dissemination of their results during the period mentioned
therein, i.e., the hour fixed for commencement of polls in the first phase and half hour
after the time fixed for close of poll for the last phase in all the States and Union
Territories.
2. (iii) Section 127A of the Representation of the People Act, 1951: The printing and
publication of election pamphlets, posters, etc. is governed by its provisions, which make
it mandatory to bear on its face the names and addresses of the printer and the
publisher.
3. (iv) Section 171H of the Indian Penal Code: It prohibits incurring of expenditure on,
inter alia, advertisement without the authority of the contesting candidate.

Hence, statement II is correct but it is not the correct explanation of statement I.

7
277
3
79. Consider the following statements:
6 06
1. Conduct of elections of the Houses of Parliament and to the House or Houses of
- 95 Legislature
the
of each State.
.c
2. Details about the structure of administrative machinery for the conduct omof elections.
ai
3. Qualifications and disqualifications for membership of those houses.
l
4 ;gm
How many of the above are the provisions mentioned 6 Representation of People Act, 1951?
#in
&
A. Only one
a95
B. Only two ta v
as
C. All three
sr iv
i
ut
D. None
r
Your Answer : A - sh
a
Cv
Correct Answer t: a
i v as
Answeri SrJustification :
t
h ru
S RPA act, 1951 provides for-
Conduct of elections of the Houses of Parliament and to the House or Houses of the
Legislature of each State.

Hence, statement 1 is correct.

Details about the structure of administrative machinery for the conduct of elections.

Hence, statement 2 is correct.

Qualifications and disqualifications for membership of those houses.

Hence, statement 3 is correct.

Corrupt practices and other offences at or in connection with such elections and the

instacourses.insightsonindia.com 55
© Insights Active Learning | All rights reserved - 18107. You may not reproduce, distribute or exploit the contents in any
form without written permission by copyright owner. Copyright infringers may face civil and criminal liability
Total Marks : 200.00
Prelims Test 22 - GS Test 15
( Insta Prelims Test Series 2024 7.0 ) Mark Scored : 83.54

decision of doubts and disputes arising out of or in connection with such elections.

80. Consider the following statements regarding Electoral Bonds:


1. The central government notified the Electoral Bond Scheme in 2018.
2. It is aimed at bringing clean money and substantial transparency into the system of political
funding.
3. The electoral bonds may be purchased by a citizen of India or entities incorporated or
established in India.

How many of the above statements are incorrect?


A. Only one
B. Only two
C. All three
7
77
D. None
3 2
6
Your Answer : A
560
Correct Answer : D - 9
m
Answer Justification : il.co
m a
Electoral Bonds: 4 ;g
6
&#
95 Electoral Bond Scheme.
In 2018, the central government notified the
t a va
Hence, statement 1 is correct. vas
i sri
t
This scheme was announced
h ruin the 2017 budget. It is touted as an alternative to cash donations
- s It is aimed at bringing clean money and substantial
made to the political parties.
a
tav the system of political funding.
transparency into
s
a
riv
Hence,Sstatement 2 is correct.
t i
h ru salient features of the scheme are:
The
S
(i) The electoral bond means a bond issued in the nature of promissory note which is a bearer
banking instrument and does not carry the name of the buyer or payee.

(ii) The electoral bonds may be purchased by a citizen of India or entities


incorporated or established in India.

Hence, statement 3 is correct.

81. Consider the following statements regarding CAG:


1. He does not hold his office till the pleasure of the president, though he is appointed by him.
2. He is not eligible for further office, under the Government of India, after he ceases to hold his
office but can hold office under states government.

instacourses.insightsonindia.com 56
© Insights Active Learning | All rights reserved - 18107. You may not reproduce, distribute or exploit the contents in any
form without written permission by copyright owner. Copyright infringers may face civil and criminal liability
Total Marks : 200.00
Prelims Test 22 - GS Test 15
( Insta Prelims Test Series 2024 7.0 ) Mark Scored : 83.54

3. The conditions of service of persons serving in the Indian Audit and Accounts Department and
the administrative powers of the CAG are prescribed by the president after consultation with
the CAG.

How many of the above statements are incorrect?


A. Only one
B. Only two
C. All three
D. None

Your Answer : B
Correct Answer : A

Answer Justification :
77
27
The Constitution has made the following provisions to safeguard and ensure the independence
3
6
of CAG: 60 5
-9
.c om
ail
1. He is provided with the security of tenure. He can be removed by the president only in
4 ;gmThus, he does not hold his
accordance with the procedure mentioned in the Constitution.
office till the pleasure of the president, though#he 6 is appointed by him.
5 &
va9
ta
i vas
Hence, statement 1 is correct.
tisr
u
s hr
v a-
sta for further office, either under the Government of India or of any state, after
2. He is not eligible
a
v hold his office.
he ceasesrito
i S
r ut
Sh
Hence, statement 2 is incorrect.

3. His salary and other service conditions are determined by the Parliament.

4. His salary is equal to that of a judge of the absence, pension or age of retirement can be
altered to his disadvantage after his appointment.

instacourses.insightsonindia.com 57
© Insights Active Learning | All rights reserved - 18107. You may not reproduce, distribute or exploit the contents in any
form without written permission by copyright owner. Copyright infringers may face civil and criminal liability
Total Marks : 200.00
Prelims Test 22 - GS Test 15
( Insta Prelims Test Series 2024 7.0 ) Mark Scored : 83.54

5. The conditions of service of persons serving in the Indian Audit and Accounts Department
and the administrative powers of the CAG are prescribed by the president after consultation
with the CAG.

Hence, statement 3 is correct.

6. The administrative expenses of the office of the CAG, including all salaries, allowances and
pensions of persons serving in that office are charged upon the Consolidated Fund of India.
Thus, they are not subject to the vote of Parliament.

7 77
82. Consider the following statements regarding Joint State Public Service Commission 3 2
(JSPSC):
6 06
5
1. A JSPSC is an extra-constitutional Body. -9
2. The chairman and members of a JSPSC are appointed by the president. m
.co
3. They can resign from their offices at any time by submitting theirilresignation letters to the
a
;gm
governors of respective states.
6 4
How many of the above statements are correct? &#
5
A. Only one va9
B. Only two
asta
C. All three
sr iv
i
D. None
r ut
- sh
Your Answer : A va
Correct Answera st: aB
S riv
i
ut Justification :
Answer
r
Sh The Constitution makes a provision for the establishment of a Joint State Public Service
Commission (JSPSC) for two or more states. While the UPSC and the SPSC are created directly
by the Constitution, a JSPSC can be created by an act of Parliament on the request of
the state legislatures concerned. Thus, a JSPSC is a statutory and not a constitutional
body.

Hence, statement 1 is correct.

The chairman and members of a JSPSC are appointed by the president.

Hence, statement 2 is correct.

They hold office for a term of six years or until they attain the age of 62 years, whichever is
earlier. They can be suspended or removed by the president. They can also resign from
their offices at any time by submitting their resignation letters to the president. The

instacourses.insightsonindia.com 58
© Insights Active Learning | All rights reserved - 18107. You may not reproduce, distribute or exploit the contents in any
form without written permission by copyright owner. Copyright infringers may face civil and criminal liability
Total Marks : 200.00
Prelims Test 22 - GS Test 15
( Insta Prelims Test Series 2024 7.0 ) Mark Scored : 83.54

number of members of a JSPSC and their conditions of service are determined by the president.

Hence, statement 3 is incorrect.

83. Consider the following statements regarding National Investigation Agency (NIA):
1. It was constituted in 1968 under the provisions of the National Investigation Agency Act, 1968
(NIA Act).
2. It is the central counter-terrorism law enforcement agency in the country.
3. The Director-General of NIA has powers similar to the powers exercisable by a Director-General
of Police in respect of the police force in a state.

How many of the above statements are incorrect?


A. Only one 7
B. Only two 277
C. All three 6 3
D. None 560
- 9
m
Your Answer : A
il.co
Correct Answer : A
m a
4 ;g
6
Answer Justification :
&#
a95
The National Investigation Agency (NIA) was v
ta constituted in 2009 under the provisions of the
National Investigation Agency Act, 2008 as(NIA Act). It is the central counter-terrorism law
r i v
enforcement agency in the country.
u tis
hr
Hence, statement 1 -issincorrect but statement 2 is correct.

t a va
The NIA was a s
established in the backdrop of the 2008 Mumbai terror attacks, popularly known as
i v
the 26/11rincident. This national horror led to the realisation of the need for a separate federal
t
agency i Sto deal with terror-related crimes in the country.
ru
Sh The NIA is headed by a Director-General. He is appointed by the central government. His
powers are similar to the powers exercisable by a Director-General of Police in respect
of the police force in a state.

Hence, statement 3 is correct.

84. Consider the following statements regarding the Central Vigilance Commission (CVC):
1. The CVC is conceived to be the apex vigilance institution, free of control from any executive
authority.
2. The CVC is a multi-member body consisting of a Central Vigilance Commissioner (chairperson)
and not more than five vigilance commissioners.
3. They are appointed by the president by warrant under his hand and seal on the

instacourses.insightsonindia.com 59
© Insights Active Learning | All rights reserved - 18107. You may not reproduce, distribute or exploit the contents in any
form without written permission by copyright owner. Copyright infringers may face civil and criminal liability
Total Marks : 200.00
Prelims Test 22 - GS Test 15
( Insta Prelims Test Series 2024 7.0 ) Mark Scored : 83.54

recommendation of a three- member committee consisting of the Prime Minister as its head,
Speaker and the Leader of the Opposition in the Lok Sabha.

How many of the above statements are correct?


A. Only one
B. Only two
C. All three
D. None

Your Answer : B
Correct Answer : A

Answer Justification :

7
77
The CVC is conceived to be the apex vigilance institution, free of control from any
executive authority, monitoring all vigilance activity under the Central Government 3 2
and
06
advising various authorities in Central Government organisations in planning, executing,
6
reviewing and reforming their vigilance work. 95 -
m
il.co
m a
Hence, statement 1 is correct. 4 ;g
6
5 &#
va9
ta
i vas
The CVC is a multi-member body consisting of a Central Vigilance Commissioner
r vigilance commissioners.
(chairperson) and not more than stwo
i
r ut
h
Hence, statement 2 issincorrect.
-
t a va
i v as
Srappointed by the president by warrant under his hand and seal on the
Theytiare
ru
Shthe Union minister of home affairs and the Leader of the Opposition in the Lok
recommendation of a three- member committee consisting of the prime minister as its head,

Sabha.

Hence, statement 3 is incorrect.

They hold office for a term of four years or until they attain the age of sixty five years,
whichever is earlier. After their tenure, they are not eligible for further employment under the
Central or a state government.

instacourses.insightsonindia.com 60
© Insights Active Learning | All rights reserved - 18107. You may not reproduce, distribute or exploit the contents in any
form without written permission by copyright owner. Copyright infringers may face civil and criminal liability
Total Marks : 200.00
Prelims Test 22 - GS Test 15
( Insta Prelims Test Series 2024 7.0 ) Mark Scored : 83.54

85. Consider the following statements regarding Co-operative Societies:


1. Only the central legislature has power to make provisions for the incorporation, regulation and
winding-up of co-operative societies.
2. The maximum number of directors of a co-operative society shall not exceed twenty-one.

Which of the above statements is/are incorrect?


A. 1 only
B. 2 only
C. Both 1 and 2
D. Neither 1 nor 2

Your Answer :
Correct Answer : A
7
Answer Justification : 277
3
6 06
Incorporation of Co-operative Societies: The state legislature may make 5 provisions for
the incorporation, regulation and winding-up of co-operative societies based - 9on the principles of
m
oparticipation
voluntary formation, democratic member control, member- economic
i l .c and
a
autonomous functioning. The maximum number of directors of a co-operative society shall not
exceed twenty-one. 4 ;gm
& #6
5 2 is correct.
a9
Hence, statement 1 is incorrect but statement
v
a sta
sr iv
i
86. Consider the following statements
r ut regarding STARS Project
h
- s scheme financially aided by the World Bank.
1. It is a Centrally sponsored
a
s t av
2. It is intended to improve governance of Eklavya schools in the North Eastern states through
a
Samgra Shiksha.
riv as the National Assessment Centre for the project under the National Education
3. PARAKHSserves
i
ut 2020.
Policy
r
Sh
How many of the above statements are correct?
A. Only one
B. Only two
C. All three
D. None

Your Answer :
Correct Answer : B

Answer Justification :
STARS Project or the Strengthening Teacher Learning and Results for States is
a World Bank led initiative to improve quality and governance of school
education.
It would be implemented as a centrally sponsored scheme under the Department

instacourses.insightsonindia.com 61
© Insights Active Learning | All rights reserved - 18107. You may not reproduce, distribute or exploit the contents in any
form without written permission by copyright owner. Copyright infringers may face civil and criminal liability
Total Marks : 200.00
Prelims Test 22 - GS Test 15
( Insta Prelims Test Series 2024 7.0 ) Mark Scored : 83.54

of School Education and Literacy, Ministry of Education.


It was launched in 2020 and is scheduled to run for period of 5 years till 2024-25.

Hence statement 1 is correct.

The project will be implemented in the six states of Himachal Pradesh, Kerala,
Madhya Pradesh, Maharashtra, Orissa and Rajasthan through the Samagra
Shiksha.
Recently the Ministry of Education and the World Bank organised a workshop on School to
Work transition under the STARS program.

Hence statement 2 is incorrect.

Around 250 million students between the age group of 6 and 17 in 1.5 million schools and
over 10 million teachers will benefit from this USD 500 million program.
7
77
The program will Strengthen Public School Education and supports the goal of providing
2
‘education for all’.
0 63
6
PARAKH will serve as a national assessment centre; it will set norms for5student
assessment and evaluation for all school boards across the country. - 9
.c om Education Policy
PARAKH has been launched as part of the implementation of National
l
ai boards regarding new
2020, that envisaged a standard setting body to advise School
assessment patterns and latest research and promote m
;g collaborations between them.
#64
Hence statement 3 is correct. 5 &
va9
ta
i vas
i sr
87. Consider the following statements
r ut
1. Globally, Indian is second
- shonly to China regarding coal reserves.
t a va and Lignite (ECL) Scheme is aimed at estimating coal resources in Non
2. The Exploration of Coal

i v as blocks.
Coal India Limited
3. The UTTAM
S r application has been launched under the ECL Scheme to monitor its progress for a
ti of ten years till 2033.
period
ru
ShHow many of the above statements are correct?
A. Only one
B. Only two
C. All three
D. None

Your Answer :
Correct Answer : A

Answer Justification :
India has the 5th largest coal Reserves and is second in coal production after China
it is among the largest importer of coal, which mainly comes from Indonesia, South Africa
and Australia.
The top five states in terms of total coal reserves in India are Jharkhand> Odisha>

instacourses.insightsonindia.com 62
© Insights Active Learning | All rights reserved - 18107. You may not reproduce, distribute or exploit the contents in any
form without written permission by copyright owner. Copyright infringers may face civil and criminal liability
Total Marks : 200.00
Prelims Test 22 - GS Test 15
( Insta Prelims Test Series 2024 7.0 ) Mark Scored : 83.54

Chhattisgarh> West Bengal > Madhya Pradesh.

Hence statement 1 is incorrect.

The Exploration of Coal and Lignite Scheme has been approved by the Ministry of Coal
with an estimated expenditure of Rs. 2980 crore for a period of 5 years from 2021-22 to
2025-26.
It will explore and estimate coal resources in the country facilitating preparation of
detailed project reports for coal mining.
The project consists of two stages viz., Promotional (Regional) exploration and Detailed
exploration in Non Coal India Limited blocks.

Hence statement 2 is correct.

UTTAM or Unlocking Transparency by Third Party Assessment of Mined Coal


7
77
Application has been launched by the Ministry of Coal in 2018 for monitoring of coal
2
63 through
quality, new coal linkage policy to ensure adequate supply of fuel to power plants
0
6
reverse auction, online coal clearance system, coal allocation monitoring5system, opening
up of commercial coal mining. -9
.c om
Hence statement 3 is incorrect.
m ail
4 ;g
6
5 &#
88. Consider the following statements regarding
v a9 Model Prisons Act 2023
1. It has been prepared by the Ministry of s ta Affairs with assistance from the Bureau of Police
Home
a
iv the Prisons act 1894.
Research and Development to replace
i sr
ut Prisoners Act 1900.
2. It incorporates provisions of the
r
- sh
3. It provides for separate accommodation for transgender and women inmates.
a training of prisoners to aid their reintegration into the society.
4. It focuses on vocational
stav
a
v the above statements are correct?
How many
S riof
A. iOnly one
r uB.t Only two
Sh C. Only three
D. All four

Your Answer : D
Correct Answer : D

Answer Justification :
Recently, the Ministry of Home Affairs has announced that it has finalised the preparation
of the Model Prisons act 2023
It aims to replace the British era Prisons act of 1894.
The Ministry assigned the task of revision of the act of 1894 to the Bureau of Police
Research and Development, who, after holding wide range of discussions with state
prison authorities and experts , finally prepared the draft.
Along with the Prisons Act 1894 , the Prisoners act 1900 and the Transfer of

instacourses.insightsonindia.com 63
© Insights Active Learning | All rights reserved - 18107. You may not reproduce, distribute or exploit the contents in any
form without written permission by copyright owner. Copyright infringers may face civil and criminal liability
Total Marks : 200.00
Prelims Test 22 - GS Test 15
( Insta Prelims Test Series 2024 7.0 ) Mark Scored : 83.54

Prisoners act 1950 have been reviewed by the Ministry and relevant provisions
of these acts have been assimilated in the new Model Prisons act 2023.
The new guidelines aim at reforming prison management and ensuring the transformation
of inmates into law abiding citizens and their rehabilitation in the society.

Hence statements 1 and 2 are correct .

The new act contains provision regarding provision for legal aid to prisoners, probation,
parole and premature release to incentivise good conduct.
It also provides for separate accommodation for women Prisoners, transgenders, etc.
The act prohibits and makes provision for punishment for use of prohibited items inside
jails for prisoners as well as jail staffs.
There is provision regarding establishment and management of high security jails, open
jails, video conferencing with courts , scientific and Technological intervention in prisons
and so on 7
2
the act focuses on vocational training and skill development of prisoners and their 77
3
reintegration into the society. 06
9 56
Hence statements 3 and 4 are correct. -
.c om
ail
4 ;gm
89. Consider the following statements regarding Startup
#6 India Seed Fund Scheme
& monetary support from any government
1. To be eligible, a startup should not have received5any
a9
tav and Industry to startups engaged in
scheme.
s
va
2. Preference is given by the Ministry of Commerce
r i
tisExpert Advisory Committee for providing grants for validation
healthcare, education and financial inclusion.
r
3. Incubators will be selected by uan
sh
of proof of concept and- prototype development.
a
st av
How many ofathe above statements are correct?
v
rione
S
A. Only
i
r uB.t Only two
Sh C. All three
D. None

Your Answer : B
Correct Answer : B

Answer Justification :
The Startup India Seed Fund Scheme was announced at Startup India International
summit in January 2021.
To be eligible under the scheme, a startup recognised by the Department for Promotion of
Industry and Internal Trade (Ministry of Commerce and Industry ) should not be
incorporated more than 2 years ago at the time of application.
Startups should not have received more than 10 lakhs of monetary support
under any Central or state government scheme.

instacourses.insightsonindia.com 64
© Insights Active Learning | All rights reserved - 18107. You may not reproduce, distribute or exploit the contents in any
form without written permission by copyright owner. Copyright infringers may face civil and criminal liability
Total Marks : 200.00
Prelims Test 22 - GS Test 15
( Insta Prelims Test Series 2024 7.0 ) Mark Scored : 83.54

Hence statement 1 is incorrect.

Preference would be given under the scheme to start ups creating innovative solutions in
sectors such as social impact, waste management, financial inclusion, education,
agriculture, food processing , biotechnology, energy, mobility, healthcare, defence, space,
railways, oil and gas, textiles and so on.

Hence statement 2 is correct.

An Expert Advisory Committee has been constituted by the Ministry to monitor overall
execution and selection of incubators for allotment of seed funds
grants of up to Rs 5 crores will be provided to eligible incubators selected by the
committee
the selected incubators will provide grants of up to Rs 20 lakhs for validation of proof of
concept, prototype development or product trials to startups
7
277
Hence statement 3 is correct. 6 3
560
- 9
m
90. The Nyaya Vikas Scheme is related to the il.co
m a
;g
A. 64subordinate courts.
Construction of court halls for judges of district and
#
B. &
Monitoring of recruitment of parliamentary5staffs
a 9
C. Supervision of Lok Adalats v
D. sta
Monitoring mental condition andamaintenance of privacy of women litigants
sr iv
Your Answer : u ti
Correct Answer : A shr
-
t a va
i v as
Answer Justification :
Ther Nyaya Vikas scheme is implemented by the department of justice and under Ministry
i S
t of law and justice
h ru It is a centrally sponsored scheme for the development of infrastructure facilities for
S
district and subordinate judiciary since 1993-94.
Under it, Central assistance is provided to the state governments and union
territory administrations for construction of court halls and residential units for
judicial officers and judges of district and subordinate courts.
With further extension of the scheme beyond 2021, some new features like lawyer halls,
toilet complexes and digital computer rooms have been added to the scheme for
convenience of lawyers and litigants besides Court halls and residential units
the fund sharing under the scheme for centre and state is 60: 40 in respect of states
other than north Eastern states for which the fund sharing is 90:10 and 100% in respect
of union territories.
Recently, Nyaya Vikas portal has been created for monitoring implementation of the
central scheme.

Hence option A is correct.

instacourses.insightsonindia.com 65
© Insights Active Learning | All rights reserved - 18107. You may not reproduce, distribute or exploit the contents in any
form without written permission by copyright owner. Copyright infringers may face civil and criminal liability
Total Marks : 200.00
Prelims Test 22 - GS Test 15
( Insta Prelims Test Series 2024 7.0 ) Mark Scored : 83.54

91. Consider the following statements


1. The Central Electricity Authority is established under the provisions of Electricity Act 2003.
2. The National Electricity Plan (NEP) is prepared in accordance with the National Electricity Policy
by the Central Electricity Authority.
3. The NEP 2022-32 envisages that the share of Non-Fossil based capacity in India is likely to
increase to more than 50% by the end of 2026-27.

How many of the above statements are correct?


A. Only one
B. Only two
C. All three
D. None

Your Answer : B 7
Correct Answer : B 277
6 3
560
Answer Justification : 9
-3 of the Electricity
The Central Electricity Authority is established under section m
Supply Act 1948.
i l .co
The functions and duties of the CEA are defined under m a 73 of the Electricity Act
section
2003. 4;g
6
&# members (including chairperson), of
The Authority consists of not more than fourteen
5
whom not more than eight are full timeato9 be appointed by the central government.
s tav
Hence statement 1 is incorrect.iva
tisr
As per section 3(4) rofu
sh the Electricity Act 2003, the Central Electricity Authority
a -
has been mandated to prepare a National Electricity Plan in accordance with
v
ta Electricity Policy and notify such policy once in every 5 years.
the National
s
a the Central Electricity Authority has notified the National Electricity Plan for the
riv of 2022-32.
Recently
i S
period
r ut
ShHence statement 2 is correct.
NEP 2022-32 envisages that the share of non Fossil based capacity is likely to
increase to 57.4% by the end of 2026-27 and may likely to further increase to
68.4% by the end of 2031-32 from around 42.5% as on April 2023.

Hence statement 3 is correct.

92. Consider the following statements regarding Mission on Advanced and High Impact
Research
1. It is jointly funded by DRDO and the World Bank.
2. It aims to achieve Net Zero emissions along with promoting the UN Sustainable Development
goals.

instacourses.insightsonindia.com 66
© Insights Active Learning | All rights reserved - 18107. You may not reproduce, distribute or exploit the contents in any
form without written permission by copyright owner. Copyright infringers may face civil and criminal liability
Total Marks : 200.00
Prelims Test 22 - GS Test 15
( Insta Prelims Test Series 2024 7.0 ) Mark Scored : 83.54

3. It will facilitate commercialisation of indigenous Technologies developed by Indian startups.


4. It will collaborate with IITs and IIMs to identify emerging technologies in the power sector.

How many of the above statements are correct?


A. Only one
B. Only two
C. Only three
D. All four

Your Answer :
Correct Answer : C

Answer Justification :
the Mission on Advanced and High Impact Research has been launched jointly
by the Ministry of Power and Ministry of New and Renewable Energy
7 77
It is aimed to identify emerging technologies and areas of future relevance for 3 2
the Global
0 6
56
power sector and to take up indigenous end to end development of relevant Technologies
. - 9
m2027-28.
the mission will run for an initial period of 5 years from 2023-24oto
a l.c
It will be funded by Ministry of Power , Ministry of New and iRenewable Energy and Central
Public Sector Enterprises under these ministries
4 ;gm
6 budgetary sources of the central
additional funding if required, will be provided from
#
government. & 5
va9
Hence statement 1 is incorrect. ta
i vas
The mission will serve tas sr
i a catalyst for National priorities such as achieving Net
r u
Zero emissions and
sh promoting initiatives like Make in India and Start Up India
a - towards achieving United Nations’ Sustainable Development goals.
it will also contribute
stav
a
riv
Hence statement 2 is correct.
i S
r ut The project will support pilot projects of indigenous Technologies developed especially by
Sh Indian startups and facilitate their commercialisation.

Hence statement 3 is correct.

MAHIR will work with premier institutions such as IITs, IIMs, NITs, IISR and universities on
one hand and public and private power sector startups and established industries with the
government acting as an enabler for creating an innovation ecosystem in the country.
It will also leverage foreign alliances and partnerships to accelerate research and
development of Advanced technologies.

Hence statement 4 is correct.

93. Consider the following statements

instacourses.insightsonindia.com 67
© Insights Active Learning | All rights reserved - 18107. You may not reproduce, distribute or exploit the contents in any
form without written permission by copyright owner. Copyright infringers may face civil and criminal liability
Total Marks : 200.00
Prelims Test 22 - GS Test 15
( Insta Prelims Test Series 2024 7.0 ) Mark Scored : 83.54

1. The Setu Bandhan Scheme aims to improve inter-state connectivity by replacing railway line
level crossings with Road over bridges.
2. Funding for the Setu Bandhan scheme is provided under the Central Road and Infrastructure
Fund under the Ministry of Finance.
3. The Central Road Fund has been created under the National Highways Act 1956 to collect cess
on vehicles running on petrol and diesel.

How many of the above statements are correct?


A. Only one
B. Only two
C. All three
D. None

Your Answer :
7
77
Correct Answer : B
3 2
6
Answer Justification :
560
The Setu Bandhan Scheme has been introduced by the Ministry of Road - 9 Transport and
Highways
.c om
i l
It has been introduced to improve inter-state connectivity specially
a in rural areas at the
borders
4 ;gm
Under it railway line level crossings are replaced6with Road over bridges and rail under
bridges in States.
5 &#
The program is intended to increase va9road safety by replacing existing level
st
crossings with bridges thus reducing a accidents at these points.
r i va
Hence statement 1 is correct.
u tis
shr
a - Minister of Road Transport and Highways provided that Setu Bandhan
Recently the Union
v
a sta
scheme approval has been granted for seven bridge projects in Arunachal Pradesh under
r iv
the Central Road and Infrastructure Fund.
S
u ti the fund was established under the Central Road and Infrastructure Fund act
r 2000 under the Ministry of Finance.
Sh Earlier the fund was under the Ministry of Road Transport and Highways.
The fund was earlier known as Central Road Fund.

Hence statement 2 is correct.

The Central Road and Infrastructure Fund act 2000 gave statutory status to the
existing Central Road Fund which was governed by a Resolution of the
Parliament passed in 1988 for development and maintenance of national highways
and improvement of safety at railway crossings.
The Central Road and infrastructure Fund, for maintenance and improvement of
highways, is provided with the power to levy and collect, by way of cess, a duty of excise
and duty of customs on petrol and diesel.

Hence statement 3 is incorrect.

instacourses.insightsonindia.com 68
© Insights Active Learning | All rights reserved - 18107. You may not reproduce, distribute or exploit the contents in any
form without written permission by copyright owner. Copyright infringers may face civil and criminal liability
Total Marks : 200.00
Prelims Test 22 - GS Test 15
( Insta Prelims Test Series 2024 7.0 ) Mark Scored : 83.54

94. ANTARDRISHTI, recently in news, is related to

A. Surveillance of malwares and ransomwares in government systems


B. Monitoring financial inclusion at grassroot levels
C. Checking misuse of public funds
D. Monitoring of child trafficking

Your Answer :
Correct Answer : B

Answer Justification :
ANTARDRISHTI is a dashboard launched by the Reserve Bank of India.
That dashboard will provide the required insight to assess and monitor the
progress of financial inclusion by capturing relevant parameters. 7
RBI has been promoting financial inclusion through various policy initiatives. 2 77
3
06 levels
the facility will also enable to assess the extent of financial exclusion at granular
6
across the country, so that such areas can be addressed.
- 95
It is presently intended for internal use in the RBI and will furthermfacilitate greater
financial inclusion through a multi-stake holder approach. l. c o
m ai
To measure the extent of financial inclusion, RBI had constructed the Financial Inclusion
;g
# 64
Index in 2021 based on the three dimensions of financial inclusion viz., Access, Usage and
Quality. &
The launch of ANTARDRISHTI is anotherastep 95 towards RBI’s to further promote financial
inclusion in the country. tav
i vas
Hence option B is correct. tisr
u
shr
-
t a va
s
95. Consider theafollowing statements regarding PM- KISAN
r i v
S for a fixed monthly income support for all small and marginal farmers with a
1. It provides
i
t
ru
cultivable land holding below two hectares.
ShState governments are responsible for identifying and preparing database of eligible
2.
beneficiaries.
3. Farmer families with member whose monthly pension is less than Rs.10000 are included under
the scheme.
4. Farmer families with members who have held constitutional posts at some point of time are
ineligible for benefits under the scheme.

How many of the above statements are correct?


A. Only one
B. Only two
C. Only three
D. All four

Your Answer :
Correct Answer : C

instacourses.insightsonindia.com 69
© Insights Active Learning | All rights reserved - 18107. You may not reproduce, distribute or exploit the contents in any
form without written permission by copyright owner. Copyright infringers may face civil and criminal liability
Total Marks : 200.00
Prelims Test 22 - GS Test 15
( Insta Prelims Test Series 2024 7.0 ) Mark Scored : 83.54

Answer Justification :
The Pradhan Mantri Kisan Samman Nidhi or PM KISAN is a central sector scheme with
100% funding from the Government of India.
Under the scheme, income support of Rs 6000 per year in three equal
installments is provided to all land holding farmer families providing them
socio-economic security.
The amount is directly transferred to the accounts of beneficiaries without any
involvement of intermediaries, making it one of the largest direct benefit transfer
schemes in the world.

Hence statement 1 is incorrect.

The states shall prepare database of eligible beneficiary landholder farmer families inthe
villages capturing name, age, gender, category, Aadhar number, voter card or any other
identification documents issued by the central/ state/ UT government. 7
Responsibility of identifying the landholder farmer family eligible for benefit 2 77
3
under the scheme shall be of the state / UT government. 06
9 56
Hence statement 2 is correct -
.c om
All institutional landholders are in eligible for benefit under the a i l
scheme. Also farmer families
with one or more of its members belonging to the following ;g m
categories are also not eligible for
4
benefit under the scheme:
& #6
5
v
Former and present holders of constitutionala9 posts
Former and present Centre or state s taministers/ members of parliament /state legislature
i a
vmunicipal
/former and present Mayors
i srof corporations/ chairpersons of district
Panchayats, etc. u t
s hrofficers and employees of Central/ state government /ministries/
All serving and retired
v a - and its field units ;Central or state PSEs
offices/ departments
a sta
All superannuated or retired pensioners whose monthly pension is Rs 10000 or more
i v
i Sr
(excluding multitasking staff /class IV/ group D) employees of above category
t all persons who paid income tax in last assessment year
h ru Professionals like doctors ,engineers, lawyers, chartered accountants and architects
S
registered with professional bodies and carrying out profession by undertaking practices.

Hence Statements 3 and 4 are correct

96. Regarding PM – PRANAM Scheme, which of the following statements is correct?

A. It aims to reduce pollution by encouraging ethanol blending in petrol


B. It promotes usage of biofertilizers and organic fertilizers
C. It encourages fortification of cereals to address malnuition among women and children
under age of five
D. it is intended to promote millet cultivation as well as processing and value addition in
India

instacourses.insightsonindia.com 70
© Insights Active Learning | All rights reserved - 18107. You may not reproduce, distribute or exploit the contents in any
form without written permission by copyright owner. Copyright infringers may face civil and criminal liability
Total Marks : 200.00
Prelims Test 22 - GS Test 15
( Insta Prelims Test Series 2024 7.0 ) Mark Scored : 83.54

Your Answer : D
Correct Answer : B

Answer Justification :
PM-PRANAM or PM Programme for Restoration, Awareness Management and
Amelioration of Mother Earth was announced in the budget 2023-24 by the Union
government.
The scheme encourages the balanced use of fertilizers along with use of
biofertilizers and organic fertilizers
It aims to reduce the use of chemical fertilizers by incentivising States to adopt
alternative fertilizers.
The scheme will be financed by savings of existing fertilizers subsidies under the schemes
run by the Department of Fertilizers , Ministry of Chemicals and Fertilizers.
The centre will provide 50% of the subsidy savings to the states as a grant, out of which
77
70% can be used to create assets related to technology adoption of alternative fertilizers
7
and production units at various levels 3 2
the remaining 30% can be used to encourage farmers and other stakeholders 6 06involved in
fertilizer reduction and awareness generation. 95 -
m
Hence option B is correct
il.co
m a
4 ;g
6
&#
97. Consider the following statements regarding
a 95SAMARTH scheme
v
taprovide training in spinning and weaving to trainees
1. Under it social sector NGOs and Trusts will
a s
sr iv
for employment in the organised sector.
i
2. The trainees should be over 18tyears of age and must belong to the ‘Person with Disability’
r u
(PWD) category as definedhunder Article 15(3) of the Constitution of India.
a -s
tavstatements are correct?
Which of the above
s
A. 1 onlyva
ri
B. i2Sonly
t
h ruC. Both 1 and 2
S D. Neither 1 nor 2
Your Answer :
Correct Answer : D

Answer Justification :
SAMARTH or the Scheme for Capacity Building in Textile Sector has been
launched by Ministry of Textiles to skill the youth for gainful and sustainable
employment in the textile sector for entire value chain of textile except
spinning and weaving in the organised sector.
It is implemented by institutions/ organisation of the Ministry of Textiles /state
governments having training in infrastructure and placement tie up with textile industry;
reputed training institutions/ NGOs/ societies / Trusts/ Organisations/ company start-ups
/entrepreneurs active in textile sector having placement tie ups with textile industry.

instacourses.insightsonindia.com 71
© Insights Active Learning | All rights reserved - 18107. You may not reproduce, distribute or exploit the contents in any
form without written permission by copyright owner. Copyright infringers may face civil and criminal liability
Total Marks : 200.00
Prelims Test 22 - GS Test 15
( Insta Prelims Test Series 2024 7.0 ) Mark Scored : 83.54

Hence statement 1 is incorrect.

The trainee must be a citizen of India possessing and Aadhar Card ( unless exempted
under Aadhaar act 2016 ) with age over 14 years.
Preference is given to marginalized social groups ( i.e., women, scheduled castes ,
Scheduled tribes, as well as handicapped persons, minorities and persons from BPL
category).

Hence statement 2 is incorrect .

98. India Climate Energy Dashboard 3.0 has been launched by

A. NITI Aayog
7
B. Ministry of Environment Forest and Climate Change
277
C. Ministry of New and Renewable Energy 6 3
D. Council on Energy Environment and Water 560
- 9
m
Your Answer :
il.co
Correct Answer : A
m a
4 ;g
6
&#
Answer Justification :
The India Climate Energy Dashboard93.0 5 has been launched by NITI Aayog
va
sta sourced from government publications
It is a comprehensive platform that provides near real time data on the energy sector,
a
climate and related economic datasets
iv allows free access and analysis of data sets using an
i
It is a user friendly platform srthat
r
analytical engine enablingut users to gain insights and better understanding climate and
h
energy sectors. - s
t a va
Hence option
i v asA is correct
i Sr
t
h ru
S
99. Consider the following statements
1. The Centralised Public Grievance Redressal and Management System (CPGRAMS) enables
citizens to launch their grievances to public authorities connected to all ministries.
2. The Grievance Redressal Assessment and Index (GRAI) 2022 is based on CPGRAMS database.
3. GRAI is published by NITI Aayog best on the two parameters of Efficiency and Service delivery.

How many of the above statements are correct?


A. Only one
B. Only two
C. All three
D. None

Your Answer : B
Correct Answer : B

instacourses.insightsonindia.com 72
© Insights Active Learning | All rights reserved - 18107. You may not reproduce, distribute or exploit the contents in any
form without written permission by copyright owner. Copyright infringers may face civil and criminal liability
Total Marks : 200.00
Prelims Test 22 - GS Test 15
( Insta Prelims Test Series 2024 7.0 ) Mark Scored : 83.54

Answer Justification :
CPGRAMS is an online platform available to the citizens 24/7 to lodge their grievances to
the public authorities regarding any matter related to service delivery.
The citizens can access it through mobile application in smartphones.
it is a single portal connected to all ministries and respective departments of the
Government of India and States
Every ministry have role based access to this system.

Hence statement 1 is correct.

The Grievance Redressal Assessment and Index 2022 has been conceptualized
and designed by the Department of Administrative Reforms and Public
Grievances, Ministry of Personnel , Public Grievances and Pension.
It provides a comparative picture and valuable insights about areas of strength and
improvement regarding grievance redressal mechanism in various ministries of the 7
government. 277
3
The latest index has been published using data between January and December
6 06 2022
from the Centralised Public Grievance Redressal and Management System 95 . -
m
Hence statement 2 is correct.
il.co
m a
4 ;g
Under GRAI, eighty nine Central ministries and departments were assessed and ranked
#6 of Efficiency, Domain, Organisational
based on a comprehensive index in the dimensions
&
commitment and Feedback and Corresponding
a 95 12 indicators.
GRAI is published by the Department
s tav of Administrative Reforms and Public
a
Grievances, Ministry of Personnel
iv
, Public Grievances and Pension.
sr
ti
ru
Hence statement 3 is incorrect
- sh
a
stav
a
r v following statements regarding Digital India Act 2023
100. Consider ithe
S
u ti
1. It provides guidelines for use of Bitcoin technology.
r
2. hIt supports the concept of Artificial Intelligence and open Internet.
S
3. Under it, ‘Know Your Customer’ requirements are mandatory for wearable devices.

How many of the above statements are correct?


A. Only one
B. Only two
C. All three
D. None

Your Answer : A
Correct Answer : B

Answer Justification :
The Digital India Act 2023 will replace the Information Technology act 2000.
The DIA 2023 recognises the changes that internet has gone since inception of the IT Act

instacourses.insightsonindia.com 73
© Insights Active Learning | All rights reserved - 18107. You may not reproduce, distribute or exploit the contents in any
form without written permission by copyright owner. Copyright infringers may face civil and criminal liability
Total Marks : 200.00
Prelims Test 22 - GS Test 15
( Insta Prelims Test Series 2024 7.0 ) Mark Scored : 83.54

2000 and provides a comprehensive legal framework to address those changes Such that
India’s regulatory landscape is in sync with the digital revolution of the 21st century.
The act recognises the growing importance of New age technologies and
supports Blockchain technology and provides guidelines for responsible
utilisation of the same.
However, legal status is not provided to cryptocurrencies including Bitcoin and
they cannot be used for banking purposes in the country.

Hence statement 1 is incorrect.

DIA 2023 also recognises the growing importance of Artificial Intelligence and provides
certain guidelines for its utilisation.
It also upholds the concept of an open internet striking a balance between
accessibility and necessary regulation to maintain order and protect users.

77
It mandates stringent ‘Know Your Customer’ requirements for wearable devices
accompanied by criminal law sanctions. 27
63
Hence statements 2 and 3 are correct. 560
- 9
m
il.co
m a
4 ;g
6
5 &#
va9
ta
i vas
tisr
u
shr
a -
stav
a
Sriv
i
r ut
Sh

instacourses.insightsonindia.com 74
© Insights Active Learning | All rights reserved - 18107. You may not reproduce, distribute or exploit the contents in any
form without written permission by copyright owner. Copyright infringers may face civil and criminal liability

You might also like